Exammaster PANCE Practice V3 Block 1

¡Supera tus tareas y exámenes ahora con Quizwiz!

A 9-year-old girl presented to her pediatrician several months ago with arthralgias, myalgias, weight loss, and morning stiffness of her joints. She has since been diagnosed with Juvenile Rheumatoid Arthritis (JRA) and has been taking nonsteroidal anti-inflammatory (NSAIDs) medications to treat the symptoms. Her parents have recently taken her to see a rheumatologist in the hopes of making sure they are considering every form of therapy for the child despite positive results from the NSAIDs thus far. Question What first-line adjunctive treatment should be recommended? 1 Addition of a salicylate 2 Continued use of NSAIDs 3 Addition of a disease-modifying antirheumatic agent 4 Addition of physical and occupational therapy 5 Addition of folic acid

Addition of physical and occupational therapy Explanation The correct answer is the addition of physical and occupational therapy. The prognosis of JRA is typically good, but it needs to be treated early and aggressively in order to prevent osteopenia, asymmetrical skeletal development, systemic manifestations, and chronic eye disease. Aggressive therapy refers to pharmacologic therapy as well as lifestyle and dietary changes. The goal of physical therapy (PT) is to strengthen muscles, decrease the impact loading on joints, and increase range of motion. Patients are given instructions for using moist heat on the joints and are taught both passive and active assisted exercises to perform 2-3 times daily. Occupational therapy (OT) is used to improve the patient's body mechanics, posture, and other modalities to protect their joints. Patients are given instructions by an occupational therapist about proper posture, improvement in activities of daily living, body mechanics, and how to protect their joints in general. It is important for patients with JRA to use exercise during all stages of their disease. During the early stages, patients should use passive range of motion in order to maintain range of motion during the most painful period. As pain is improved and inflammation decreases, patients should slowly increase to active forms of exercise. Salicylates can be used as first-line treatment for patients with JRA, but would not be used in conjunction with an NSAID. NSAIDs have become preferred over salicylates in recent years due to the decreased frequency of dosing, their liquid form, and diminished side effects. While NSAIDs are commonly used as a first-line agent and the patient has shown improvement, JRA is a disease that has historically shown better results with early and aggressive treatment. Aggressive treatment refers to the use of a combination of various forms of treatment, such as pharmacologic treatments, in conjunction with lifestyle modifications. Disease-modifying antirheumatic agents (DMARDs) are considered a second-line form of treatment for patients with JRA unless the patient has contraindications to NSAIDs and salicylates. DMARDs are not given in conjunction with an NSAID but may be used as combination therapy; most commonly, 2 DMARDs are used together. Combination therapy is the first-line treatment in patients with erosions on radiograph or other signs of more aggressive disease at the time of diagnosis.

A 3-year old boy is brought to the pediatrician with a 2-week history of symptoms of an itchy, red "wound" on the right knee. The mother ignored the wound initially, assuming that the boy got injured while playing on the street, but it did not heal and seems to have worsened. On examination, there are a few intact vesicles and a few ruptured vesicles covered with honey-colored crusts. You suspect non-bullous impetigo. Question What treatment will you prescribe? 1 Antibiotic ointment 2 Steroid ointment 3 Emollient cream 4 Oral antibiotics 5 Oral steroids

Antibiotic ointment Explanation Impetigo is a contagious pyogenic infection classified into bullous and non-bullous types. The non-bullous type is caused by Staphylococcus aureus or Streptococcus pyogenes and is more common on the exposed parts of the body. An initial breach in skin barrier by trauma, insect bite, etc., occurs, followed by infection. It is most common in the age group 2-5 years and in those living in poor hygiene or overcrowded settings. It is a self-limiting condition and treatment may aid in reducing transmission and relieving discomfort. Treatment is with proper wound care and topical antibiotics like mupirocin or retapamulin. The bullous type is characterized by large bullae, along with fever, diarrhea, weakness, etc. Topical or oral steroids are not indicated in skin infections. Oral antibiotics are usually not necessary in non-bullous impetigo, although they may be indicated in extensive bullous lesions. Emollients are not necessary in impetigo.

A 65-year-old man with a past medical history of hypertension, chronic arthritis, and herniated lumbar discs presents with a 1-week history of severe, tearing pain to his rectal area that occurs while he defecates. Following his bowel movement, he notes relief of the tearing pain, but he feels a throbbing sensation in the area. He has noted small drops of bright red blood on the toilet paper. He presently takes amlodipine and hydrochlorothiazide for hypertension as well as hydrocodone-acetaminophen for pain. He has noticed a reduced frequency of bowel movements of late due to pain. The physical examination is notable for a 5-mm midline crack-like lesion at the anus in the 6 o'clock position. The remainder of the physical exam is unremarkable. Question What is the most likely diagnosis? 1 Internal hemorrhoid 2 Anal fissure 3 Perianal fistula 4 Gonococcal proctitis 5 Pilonidal cyst

Correct Answer: Anal fissure This patient's most likely diagnosis is an anal fissure. These lesions are linear or rocket-shaped ulcers that are usually less than 5 mm in length and arise from trauma due to defecation. Underlying contributory conditions may include straining, constipation, or high internal sphincter tone. They occur most commonly in the posterior midline and appear as cracks in the epithelium; those that do not occur midline should raise suspicion for underlying Crohn's disease, HIV/AIDS, tuberculosis, syphilis, or anal carcinoma. Commonly, patients present with a severe, tearing pain during defecation followed by a throbbing discomfort. Often, constipation occurs due to fears of recurrent pain. There may also be hematochezia. Pain is unusual with internal hemorrhoids. Gonococcal proctitis may cause itching, burning, tenesmus, and a mucopurulent discharge. Perianal fistulas are typically preceded by abscesses, in which there is constant perianal pain and throbbing. Fistulas cause purulent discharge that may lead to itching, tenderness, and pain. A pilonidal cyst is a fairly common, usually asymptomatic, opening of a sinus tract located in the midline superficial to the coccyx or the lower sacrum. This opening may exhibit a small tuft of hair surrounded by a halo of erythema. The cyst may occasionally have slight drainage and may be associated with abscess formation and secondary sinus tracts.

You are evaluating a 14-year-old African American girl. She is a high school sophomore and a standout on her school's tennis team. She is going to attend a 2-week tennis camp during the summer break. She states that she always becomes more easily fatigued during the summer months, despite adequate hydration. Although she would never consider using any drugs that would increase her energy level, she has begun consuming a variety of energy drinks on the advice of her older teammates. Her favorite contains caffeine (110 mg/8 oz) and guarana (30 mg/8 oz), among other ingredients. She consumes approximately three 16 oz cans per day on heavy practice days and during competition. While her energy levels have increased, she occasionally notices some jitteriness, a rapid heart rate continuing well after cessation of exercise, and a feeling of tiredness about 4 hours after consuming the drink. She wants to know if she should continue consuming the energy drink during summer camp. Past medical history is unremarkable and there is no history of tobacco, alcohol, or street drug use. Growth and development have been normal and immunizations are current. Vital signs are normal except for a resting pulse of 110. The examination is otherwise unremarkable, consistent with Tanner stage IV. Question How do you respond? 1 "It is especially helpful to consume energy drinks during hot weather." 2 "You should discontinue the use of all energy drinks." 3 "You should switch to an energy drink without guarana." 4 "You should limit your energy drink consumption to every other day." 5 "Energy drinks do not really provide an energy burst."

Correct Answer: "You should discontinue the use of all energy drinks." Explanation Energy drinks should be completely eliminated from her regimen. Energy drinks are touted as giving a quick burst of energy, increased mental alertness, and decreased fatigue. They are transiently effective in producing these results. Their components are largely a variety of stimulants, especially caffeine, but also guarana, a plant from South America with a caffeine-containing bean. Energy drinks are to be distinguished from sports drinks, which contain carbohydrates, electrolytes, minerals, and sometimes vitamins. While sports drinks do have a role in rehydration of athletes in the pediatric age group, energy drinks, while effective, should never be employed by children or teenagers. The actual ingredients and amounts are sometimes dubiously listed in the product labeling, but the amount of caffeine contained in a 16 oz can is such that consuming several drinks in a short period can cause tachycardia, hypertension, insomnia, and polyuria. Drinks containing guarana contain additional caffeine, frequently not included in the total caffeine content in the product labeling. Energy drinks are especially dangerous in hot weather, in part due to the diuretic effect of caffeine and increase in core temperature.

A 32-year-old man with no significant past medical history presented with dyspnea, palpitations, feelings of anxiety, and dizziness, all of which occurred earlier in the morning following a brisk walk. He denied any prior episodes, illicit drug use, alcohol or cigarette use, skipping meals, or caffeine intake. He further denied fever, chills, chest pain, history of murmurs, cough, edema, rashes, syncope, headache, psychiatric, or focal neurological complaints. The physical examination demonstrated a fast, regular pulse with a constant-intensity first heart sound, but was otherwise normal. An EKG was performed, which revealed a short PR interval plus a slurred upstroke at the beginning of the QRS complex. What is the most likely mechanism responsible for this patient's presentation? 1 Conduction delay in the proximal part of the right or left branches 2 Pre-excitation occurring via an atrio-His bundle 3 Spontaneous ectopy from muscular sleeves of pulmonary veins 4 Early excitation due to accessory pathways between the atria and ventricles 5 Inappropriately enhanced automaticity of sinus node pacemaker cells

Early excitation due to accessory pathways between the atria and ventricles Explanation Patients with Wolff-Parkinson-White syndrome have an additional aberrant muscular or nodal tissue connection (bundle of Kent) between the atria and ventricles. This conducts more rapidly than the slowly conducting AV node, and one ventricle is excited early. The manifestations of its activation merge with the normal QRS pattern, producing a short PR interval and a prolonged QRS deflection slurred on the upstroke, with a normal interval between the start of the P wave and the end of the QRS complex ("PJ interval"). The QRS complexes show an abnormal morphology with a width greater than the baseline QRS complex (often >0.11 second), with the characteristic initial slurring referred to as a delta wave. The paroxysmal atrial tachycardias seen in this syndrome often follow an atrial premature beat. This beat conducts normally down the AV node but spreads to the ventricular end of the aberrant bundle, and the impulse is transmitted retrograde to the atrium. A circus movement is thus established. Less commonly, an atrial premature beat finds the AV node refractory but reaches the ventricles via the bundle of Kent, setting up a circus movement in which the impulse passes from the ventricles to the atria via the AV node. Conduction delay in the proximal part of the right or left branches will cause a bundle branch block, not tachycardia. Rapidly firing ectopic foci in pulmonary veins have been shown to be the underlying mechanism of most paroxysmal atrial fibrillation. Pre-excitation occurring via an atrio-His bundle bypasses the AV node slow conduction area and therefore does not modify the QRS complex morphology (unlike the EKG findings in this patient). Inappropriate (nonparoxysmal) sinus tachycardia is an incessant and symptomatic tachycardia caused by inappropriately enhanced automaticity of sinus node pacemaker cells. Characteristics include: an atrial rate 100 beats/min at rest or with minimal activity an inappropriate chronotropic response with a rapid rise in heart rate with mild exertion a normal P wave axis and morphology during tachycardia absence of an underlying cause

A 67-year-old man presents for a routine follow-up visit. His past medical history includes chronic obstructive pulmonary disease (COPD) for the past 3 years and hypertension (HTN) for the past 5 years. Daily medications include a tiotropium inhaler and hydrochlorothiazide. He has smoked a pack of cigarettes a day for 50 years, and he has no desire to quit at this time. Physical exam remains unchanged from 3 months ago, and the patient has no complaints. His last pneumococcal and influenza vaccinations were 2 years ago. Question What should be offered at this visit? 1 Bupropion 2 Influenza vaccine 3 Fluticasone and salmeterol 4 Nicotine patch 5 Pneumococcal vaccine

Influnza Explanation Chronic obstructive pulmonary disease (COPD) affects approximately 32 million Americans and is the fourth leading cause of death in the US. The main cause of COPD is smoking. For this reason, COPD is considered a preventable disease. Treatment of COPD is aimed at limiting exacerbations and progression of disease. Smoking cessation is an important component of the treatment of COPD, but it will only be effective if the patient is willing to attempt to quit. Nicotine patches or gum can be used as nicotine replacement therapy to aid in smoking cessation. The dose of nicotine is gradually decreased. Bupropion (Zyban) is an antidepressant that can be used in conjunction with behavioral therapy to help in smoking cessation. Pharmacological treatment of stable COPD includes treating the airflow restriction and inflammation. Inhaled bronchodilators, such as beta-agonists and anticholinergic agents, are the mainstay of therapy. Combination therapy of inhaled beta-agonist and corticosteroids or inhaled anticholinergic and corticosteroids are indicated when a patient's symptoms are not controlled by monotherapy alone. In this case, the patient is controlled on tiotropium, so the addition of fluticasone and salmeterol (corticosteroid and long-acting beta-agonist) is not warranted. Pulmonary infections can lead to a COPD exacerbation, so patients should routinely receive pneumococcal and influenza vaccines. The CDC recommends adults receive 1 dose at age 65 years. In the case presented, the patient had his vaccine at age 65 and does not need another one. The current recommendation for the influenza vaccine is that everybody receives an annual dose, so this patient should be given an influenza vaccine at this visit.

A 72-year-old man presents with an 8-month history of progressive dyspnea, accompanied by a dry and persistent hacking cough. While the dyspnea now occurs at rest, he denies fever, chills, palpitations, chest pain, or peripheral edema. He states that he has worked for many years at a local chemical plant. His physical exam is remarkable for digital cyanosis and clubbing, while his pulmonary exam reveals diffuse, fine, dry inspiratory crackles. His cardiac exam was positive for a prominent pulmonary valve closure sound (P2) and an elevated jugular venous pressure of 6 cm. A chest X-ray noted small lung volumes, with increased densities in the lung periphery and a honeycombing pattern; pulmonary function testing measured reductions in TLC, FEV1, and FVC with a preserved FEV1/FVC ratio. What is the best treatment for this patient at this time? 1 Erythromycin 2 Furosemide 3 Lung transplantation 4 Prednisone 5 Cyclophosphamide

Prednisone Explanation This patient most likely has hypersensitivity pneumonitis (HP), also known as extrinsic allergic alveolitis. This is an inflammatory disorder of the lung involving alveolar walls and terminal airways that is induced by repeated inhalation of a variety of organic agents. The chronic form of HP typically results from low-grade or recurrent exposure over many months/years, and the lung disease may already be partially or completely irreversible. These patients are usually advised to avoid all possible contact with the offending agent. In addition to identifying and avoiding the causative agent, institution of glucocorticoid treatment is indicated. Prednisone at a dosage of 1 mg/kg per day or its equivalent is continued for 7-14 days, then tapered to 0.25-0.5 mg/kg and is maintained at this level for an additional 4-12 weeks at a rate that depends on the patient's clinical status. While patients with chronic HP may gradually recover without therapy following environmental control, a trial of prednisone may be useful to obtain maximal reversibility of the lung disease. Following initial prednisone therapy (1 mg/kg per day for 2-4 weeks), the drug is tapered to the lowest dosage that will maintain the functional status of the patient. Many patients will not require or benefit from long-term therapy if there is no further exposure to the antigen. Improvement of lung function may continue over a few months to years.

A 26-year-old woman, gravida 2 para 2, with a past medical history of Chlamydia cervicitis and recurrent vulvovaginitis has given birth to her third child via cesarean section. She is status-post delivery 24 hours, and the duration of labor was 20 hours. She presently reports that she feels "warm and feverish," has a malodorous vaginal discharge, and has a 6-hour history of achy lower abdominal pain. She denies dysuria, pyuria, urinary frequency, change in urinary odor or color, shortness of breath, chest pain, hemoptysis, nausea, vomiting, and diarrhea; she also denies any drug allergies. Her physical exam is remarkable for a fever of 100.9°F with the remainder of her vital signs normal. Additionally, she has a soft, tender lower abdomen that has noted swelling and frank pain in the lower abdomen/pelvis; a foul, white lochia is noted. The cesarean section incision site is well approximated and without induration, discharge, or erythema; there is no costovertebral tenderness, adnexal mass, or peripheral edema appreciated. What is the most appropriate first-line treatment for this patient at this time? 1 Oral trimethoprim-sulfamethoxazole DS 2 Parental clindamycin plus gentamicin 3 Parental ampicillin 4 Parental dicloxacillin 5 Parental ciprofloxacin plus doxycycline

Parental clindamycin plus gentamicin Explanation This patient's presentation is most consistent with endometritis. Established risk factors include prolonged rupture of the membranes (>24 hours), chorioamnionitis, an excessive number of digital vaginal examinations, prolonged labor (>12 hours), toxemia, intrauterine pressure catheters (>8 hours), fetal scalp electrode monitoring, preexisting vaginitis or cervicitis, operative vaginal deliveries, cesarean section, intrapartum and postpartum anemia, poor nutrition, obesity, low socioeconomic status, and coitus near term. Most puerperal infections are due to anaerobic streptococci, gram-negative coliforms, Bacteroides spp., and aerobic streptococci. In up to 30% of culture isolates, group B streptococci are partly or wholly responsible, while gram-negative species (E. coli) comprise up to 36% of all isolates. Common manifestations include fever (early maternal post-operative fever suggests infection of the womb), fever, a soft and tender uterus, cervical motion tenderness, and foul lochia. High fever and hypotension shortly after delivery are classically reflective of group B streptococci. Upon laboratory assessment, a leukocytosis above the established baseline (leukocytosis is a normal finding during labor and the immediate puerperium) indicates infection. Urinalysis and culture may indicate pyuria and bacteria in a UTI. Additional diagnostic tests include a Gram stain and wet mount of vaginal discharge or endocervical culture or DNA probe for Chlamydia. Chlamydia is more frequently associated with mild late-onset endometritis, so cultures for this organism should be obtained from patients with endometritis diagnosed several days after delivery. A combination therapy with clindamycin and an aminoglycoside is considered the criterion standard by which most antibiotic clinical trials are judged. A combination regimen of ampicillin, gentamicin, and metronidazole provides coverage against most of the organisms that are encountered in serious pelvic infections. Doxycycline should be used if Chlamydia is the cause of the endometritis. The combination of clindamycin and gentamicin administered intravenously every 8 hours has been considered the criterion standard, first-line treatment. Parental cefoxitin, Timentin, imipenem, or Unasyn with doxycycline is an alternative. In teenagers, postabortion endometritis may be caused by organisms that cause PID; the initial treatment regimen in these patients is IV cefoxitin and doxycycline. Although urinary tract infections are often associated with a clinical picture similar to that of mild endometritis, the lack of urinary complaints reliably excludes UTI, making the use of Bactrim DS inappropriate. Staphylococcus aureus is not commonly seen in cultures from women with postpartum infections of the uterus, so dicloxacillin is not a suitable empiric choice. High-dose ampicillin is effective against Group D streptococci strains (e.g., Streptococcus faecalis); group D streptococci are, however, isolated from cultures in only 14%. It is added when the patient has a less than adequate response to the usual regimen, particularly if Enterococcus spp. are suspected.

A 17-year-old boy was in your clinic 4 days ago for evaluation of a 101.8°F fever and was diagnosed with acute pharyngitis. You prescribed penicillin VK 250 mg TID for 10 days. The patient returns today because his sore throat is now worse. He has not been able to drink fluids and he has excruciatingly severe pain with swallowing. You recognize the muffled "hot potato" voice. On re-examination, you identify a right medial deviation of the soft palate with a 4+ right tonsillar swelling. Question What is the most likely diagnosis? 1 Oral candidiasis 2 Peritonsillar abscess 3 Laryngitis 4 Mononucleosis 5 Dental abscess

Peritonsillar abscess Explanation This patient is clearly suffering from a peritonsillar abscess. This occurs when an active infection penetrates the tonsillar capsule and then involves the surrounding tissue. These patients will have a severe sore throat, odynophagia, trismus, deviation of the soft palate, and an abnormally muffled voice (hot potato voice).

A 67-year-old male post office worker presents with a 3-week history of increasing shortness of breath; it occurs even while he is at rest. The patient was diagnosed with congestive heart failure in the past year and he has been well controlled on oral medication. He has no history of tobacco use. He has gained 10 pounds since his last exam 2 months prior to presentation. On physical exam, there are diminished breath sounds and decreased tactile fremitus bilaterally at the base of the lungs. Dullness to percussion is also noted in the same area. He has 3+ bilateral pitting lower extremity edema. Question Based on the patient's physical exam and history, what is the most likely diagnosis? 1 Lung malignancy 2 Tuberculosis 3 Empyema 4 Spontaneous pneumothorax 5 Pleural effusion

Pleural effusion This patient has a history of congestive heart failure, which is one of the most common causes of transudative pleural effusions. Pleural effusions in the setting of heart failure are usually right-sided or bilateral; in this patient's case, physical exam findings point to bilateral pleural effusion. Dyspnea is a common symptom seen in patients with a pleural effusion. Based on his pitting edema on exam, it is clear that this patient has uncontrolled heart failure. The physical exam findings are also common with pleural effusion. Lung malignancy is incorrect because he does not have a history of tobacco use; and because of his career as a post office worker, he has a low probability of being exposed to asbestos. Tuberculosis is incorrect because pleural effusions caused by tuberculosis are typically unilateral. A patient with TB would have a productive cough, night sweats, and weight loss as presenting symptoms. Empyema is incorrect because a patient would more than likely present with fever, productive cough, and pleuritic chest pain. A patient with empyema, however, would have diminished breath sounds and decreased tactile fremitus. Empyema is more likely to cause a massive pleural effusion with mediastinal shift. Spontaneous pneumothorax is incorrect because it is unilateral; also, the patient reports dyspnea and chest pain localized to the side of the pneumothorax. It is also acute onset. Physical examination shows hyperresonant percussion and decreased tactile fremitus localized to the side of the pneumothorax.

A 42-year-old man with a known past medical history of schizophrenia has begun to demonstrate new and unusual behavior over the past 2 weeks. The patient has been compliant with taking risperidone 2 mg for the past year. The patient's son reports that his father has been acting "silly" and exhibiting inappropriate behavior, such as removing his clothes, repeating odd noises and gestures, and speaking incoherently with random loud and violent outbursts. This morning, the patient was observed to have used a knife to cut himself; his son stopped him. Question What is next most appropriate step in the management of this patient at this time? 1 Increase the risperidone to 4 mg per day. 2 Draw a CBC, chemistry panel, and urinalysis. 3 Refer the patient to the hospital for inpatient monitoring. 4 Discontinue risperidone and begin quetiapine. 5 Prescribe both diazepam and paroxetine.

Refer the patient to the hospital for inpatient monitoring. xplanation Erythromycin ophthalmic ointment applied 2-4 times daily is a treatment option for non-sexually transmitted bacterial conjunctivitis. If trying to cover all bacterial etiologies of conjunctivitis, then erythromycin can be given in the oral form in order to include good coverage for both gonococcal conjunctivitis and chlamydial conjunctivitis. If the erythromycin ophthalmic ointment were to be prescribed in a patient with a sexually transmitted bacterial conjunctivitis, there may still be a partial or complete resolution of symptoms. Erythromycin, in either the topical or oral form, has a good chance of treating any bacterial cause of conjunctivitis until the culture results confirm the etiologic agent. Tetracycline 250 mg po 4 times daily for 3 weeks is a good treatment choice for chlamydial conjunctivitis if the cultures reveal this as the cause, but it would not be a good prophylactic choice while waiting for lab results. Bacitracin ophthalmic ointment applied 2-4 times daily for 5 days is a good treatment option for patients with bacterial conjunctivitis that is not from a sexually transmitted disease. In these cases, the most common etiologic agent is Staphylococcus aureus. Olopatadine is an antihistamine ophthalmic solution that is used in the treatment of ocular itching associated with allergic conjunctivitis. It would not be of any help in a patient with bacterial conjunctivitis, regardless of the etiology.

A 69-year-old man with a 50 pack-year smoking history, COPD for 12 years, and a myocardial infarction 2 years ago has been experiencing increased exertional dyspnea for 4 months. There is associated easy fatigability, exertional chest discomfort, and lightheadedness. He denies fever, chills, palpitations, cough, wheezing, abdominal pain, nausea, vomiting, and diarrhea. Physical exam findings were remarkable for a right ventricular heave, widely split S2 with an accentuated pulmonic component, a pulmonary ejection click, an S3 and 1+ pitting edema to the bilateral lower extremities. There was also evidence of a 5-cm jugular vein distention. What diagnostic test results would be expected in this patient? 1 Underdevelopment of central pulmonary arteries and hyperemic lung field on chest X-ray 2 Right axis deviation, R wave greater than S wave in V1, and peaked p-waves on EKG 3 A normal FEV1 to FVC ratio on pulmonary function testing 4 Increased thickness of the left atrium on echocardiography 5 Pulmonary arterial pressure of 15 mm Hg on pulmonary artery catheterization

Right axis deviation, R wave greater than S wave in V1, and peaked p-waves on EKG Explanation This patient's presentation is consistent with secondary pulmonary arterial hypertension. Common ECG findings include right axis deviation, R > S in V1, S1Q3T3, and peaked P waves. Common chest X-ray findings include enlarged hilar vessels that rapidly prune into the periphery, enlargement of central pulmonary arteries, attenuation of peripheral vessels, oligemic lung fields, and a right ventricle that fills the anterior airspace on lateral view. Given this patient's prior history of COPD, an obstructive pulmonary function test pattern is expected, not normal FEV1 to FVC ratios. Echocardiography reveals signs of chronic right ventricular pressure, such as increased thickness with paradoxical bulging of the septum into the left ventricle during systole. In later stages, right ventricular dilatation occurs, leading to hypokinesis. Right atrial dilatation and tricuspid regurgitation are also present. A mean pulmonary arterial pressure of >25 mm Hg in the absence of an underlying disorder identifies pulmonary hypertension (most patients with pulmonary arterial hypertension present with substantially higher pressures).

An adolescent girl who is established to your practice has come in to see you because of "blue areas on her skin." Over the past 5 days, the patient has noticed small red bumps that then formed areas with blue centers. Some even have blisters in the center. The patient states that she feels like she looks like a target. Examination of the patient reveals multiple lesions with central bluish discoloration as well as 3 concentric circles of color change in each lesion. While completing the physical examination, the patient states she was started on Bactrim by an urgent care physician 6 days ago to treat a urinary tract infection. Question What is the most likely diagnosis? 1 Erythema nodosum 2 Erythema multiforme 3 Pityriasis rosea 4 Psoriasis 5 Seborrheic dermatitis

Seborrheic dermatitis Explanation The scenario above is describing a patient with erythema multiforme (EM). This is a common, usually acute, and potentially recurrent inflammatory disease. Causes may include herpes simplex, upper respiratory tract infections, contact allergens, medications, and even connective tissue disorders to list a few etiologies. The patient's treatment course of Bactrim is highly likely to be the cause of her EM. Typically, the appearance of EM is numerous lesions that are described as target lesions; a confirmed diagnosis of EM should only be made if these lesions are seen. Target lesions, or "iris" lesions, originate from a centrifugal spread of the red maculopapular to a circumference of 1-3 cm in a span of 24-48 hours. The center can appear dark red, purpuric, or a dusky blue hue color. Lesions will appear in crops, and they typically resolve in a 1-2 week span without evidence of scarring, although it can take up to 1 month to completely resolve; reoccurrence is also possible. Erythema nodosum (EN) presents as pink-to-dusky-red firm nodules with indistinct edges. These nodules occur symmetrically, specifically on the pretibial surfaces; they may also be seen on the head, neck, torso, arms, and other areas. Both ankle edema and leg pain are also commonly seen with EN. Pityriasis rosea is a dermatologic disorder characterized initially by a herald patch on the trunk, followed by numerous smaller lesions of the same characteristics. A herald patch is described as an oval plaque that is usually 1-2 cm in diameter and has a thin collarette of residual scale inside its border. These lesions are salmon-colored in appearance. Psoriasis is usually described as red, sharply defined, scaling papules that coalesce to form stable round-to-oval plaques. It has a deep rich red color to it and the scale is adherent to this area, having a silvery-white appearance; when the scale is removed, it results in bleeding points (known as Auspitz sign). Areas of seborrheic dermatitis are moist, transparent (to almost yellow in color), greasy, and scaling. There are coalescing red patches and plaques.

You are evaluating a 16-year old Caucasian male high school junior (his baseball team's starting shortstop) for left hamstring soreness 4 days in duration. He felt a slight pull during infield practice. He was late that day and skipped his normal warmup and stretching routine. Despite rest, the soreness has persisted. He would like some medication for the discomfort but noted that several oral analgesics and anti-inflammatories had caused gastric upset in the past. Past medical history is otherwise unremarkable, growth and development are normal, and immunizations are current. Vital signs are normal. Examination reveals mild tenderness to palpation of the left hamstrings, aggravated by extension at the knee. Question What is the best choice of drug therapy? 1 Intramuscular methylprednisolone 2 Oral hydrocodone/acetaminophen 3 Oral prednisone 4 Topical diclofenac gel 5 Oral ibuprofen

Topical diclofenac gel Pharmacological management of minor sports injuries such as strains and sprains with topical NSAIDs provides a viable alternative to oral medication. Topical NSAIDs are generally effective in relieving the pain of minor sprains and strains when applied to the affected area and are well-tolerated, even by those who have experienced unacceptable side effects to oral NSAIDs, such as gastrointestinal upset. Topical NSAIDs have anti-inflammatory, antipyretic, and analgesic qualities. Side effects are generally limited to local site reactions, such as erythema and pruritus.

A 26-year-old woman presents with an abnormal lump in her right breast. Physical examination demonstrates a well-circumscribed, palpable mass in the upper outer quadrant of the right breast. The patient denies a family history of breast carcinoma or any additional medical problems.What would be the best test to order for further evaluation of this palpable abnormality? 1 Bilateral mammography 2 Right breast MRI 3 Right galactogram 4 Stereotactic biopsy 5 Ultrasound

Ultrasound Explanation In women under 30 with a palpable lump in their breast, the initial imaging modality should be an ultrasound. Ultrasound is an excellent modality in this scenario because it can define the borders of the mass and determine whether the lesion is solid or cystic. If the lesion is solid, then the characteristics of the mass can be better defined before either following the lesion or proceeding to biopsy. Bilateral mammography is not indicated in this patient (who is under 30) at this point. If the patient were over 30 with a palpable breast lump, then a unilateral mammogram and focused ultrasound would be the best tests to order for further evaluation. However, in patients under 30, ultrasound is a better choice for the initial test because most of these "lesions" are fibroadenomas. Additionally, it is desirable to limit radiation exposure in patients under 30, supporting ultrasound as the best choice in this case. Breast MRI exams can be used to evaluate for lobular cancer; occult breast carcinoma; to define the extent of disease in a patient with a positive surgical margin; to evaluate for multifocal, multicentric, or bilateral cancers; and to evaluate for postoperative scar versus tumor recurrence. Breast MRI is not the initial test that should be ordered to evaluate a 26-year-old patient with a palpable breast mass. Galactograms are types of mammograms whereby a breast duct is accessed using a blunt needle to evaluate for unilateral or bilateral bloody nipple discharge, which is most often caused by an intraductal papilloma but could be caused by intraductal papillary carcinoma or invasive ductal carcinoma. Galactograms would not be the initial test in the workup of a palpable mass in this young patient. A stereotactic biopsy is performed to obtain a tissue sample after a suspicious mass or cluster of microcalcifications is found on either a diagnostic or screening mammogram. A stereotactic biopsy would not be performed before an ultrasound is ordered in this patient. If a suspicious mass is seen in ultrasound, then an ultrasound-guided biopsy would be the next best course of action in order to exclude a breast carcinoma.

An 18-year-old man presents with progressive, severe knee pain. On physical examination, you note focal soft tissue swelling and erythema over the distal metaphysis of the left femur. Plain film radiographs of the left knee are obtained. Refer to the image. Question To what anatomic location does the neoplasm demonstrated on the radiograph most commonly metastasize? (xray of bones show osteosarcoma lesion on tibia ) 1 Axial skeletal red marrow 2 Contralateral knee 3 Brain 4 Lungs 5 Liver

lungs Explanation Classic osteosarcoma is the most common malignant bone tumor in the pediatric population, and it is thought to arise from primitive mesenchymal, bone-forming cells. Classically, osteosarcoma is located in a metaphyseal, intra-medullary location, with its histologic hallmark being the production of a malignant osteoid. Plain film radiographs most commonly demonstrate an aggressive, bone-forming lesion located about the knee, most often affecting the femoral or tibial metaphysis. The three most common sites of origin for osteosarcoma are the femur (40%), tibia (20%), and proximal humerus (10%). The mainstay of osteosarcoma therapy is surgical removal of the malignant lesion, with a special emphasis on limb-sparing, limb-preserving procedures that preserve function and locomotion. Metastasis to the lungs is the most common; metastases to other sites are extremely rare.

A 35-year-old man presents with a groin mass. The patient states that the mass is painless; there is no known trauma to the region. The mass is present when the patient stands, and it disappears when the patient is lying flat. Past medical history is significant for obesity, hypertension, and hyperlipidemia. Past surgical history is significant for lipoma removal from the left shoulder. The patient denies tobacco use; he tells you that he drinks about 6 beers per week. In office ultrasound confirms diagnosis and surgical repair is scheduled. Question What is the most significant risk factor for this patient's condition? 1 Obesity 2 Hypertension 3 Hyperlipidemia 4 Lipoma removal 5 Alcohol use

obesity Explanation The patient has an inguinal hernia. Hernias are more likely to occur in conditions in which there is excess pressure on the abdomen. Obesity increases abdominal pressure, so it increases the risk of hernia. Neither hypertension nor hyperlipidemia is associated with an increased risk of hernia development. While previous surgery to the abdomen may increase the risk of hernia, surgery on the shoulder (lipoma removal) would not. Tobacco use increases the risk for hernia, but alcohol use does not.

A 63-year-old woman presents for her yearly exam. She has been feeling well and does not have any current issues. She has past medical history of hypertension and hyperlipidemia. Routine laboratory testing reveals leukocytosis with 10% blasts. There is mild anemia, and platelets are normal. Peripheral blood cytometry confirms the diagnosis of chronic lymphocytic leukemia. Question: Txt at this time? 1 Imatinib 2 Lenalidomide 3 Prednisone 4 Stem cell transplant 5 Observation

observe the lady Explanation Observation is correct. Chronic lymphocytic leukemia (CLL) is a malignancy that causes the accumulation of B lymphocytes. It typically follows an indolent course. Treatment is only required for patients who are symptomatic, have extremely elevated WBC counts (greater than 100,000), or have severe anemia or thrombocytopenia. If treatment is required, commonly used medications include fludarabine, rituximab, and cyclophosphamide. Imatinib is more commonly used in the treatment of chronic myeloid leukemia (CML). Lenalidomide is usually employed in the treatment of multiple myeloma. Prednisone is sometimes used in the treatment of CLL. This patient is asymptomatic and does not require treatment. Stem cell transplant may be used in the treatment of CLL, but this is typically reserved for patients who are unresponsive to standard treatment.

A 55-year-old man presents for a follow-up on recent cholesterol tests. He has a past medical history of 2 coronary stents placed 3 days ago after presenting to the emergency department with chest pain and a 40 pack-year history of smoking (he quit smoking 2 years ago). He was not on any medications when he presented to the ER. Cholesterol tests reveal: total cholesterol 200 mg/dL; LDL 100 mg/dL; HDL 40 mg/DL; triglycerides 395 mg/dL. Question What is the most appropriate treatment to reduce future cardiovascular risk in this patient? 1 Fibrates 2 Fish oil 3 Nicotinic acid 4 Plasmapheresis 5 Statin

statin Explanation In patients with mild to moderate hypertriglyceridemia (150-500 mg/dL), the main indication for therapy is reduction of cardiovascular (CV) risk. Statins are the best therapy for CV risk reduction, and although they are not the most effective medication for reducing elevated triglycerides, they should be the first-line therapy for a patient with a high CV risk. Many studies have shown that fibrates, nicotinic acid, and fish oil can lower triglyceride levels, but they do not lower the cardiac risk as much as statins. This patient is a high-risk cardiac patient because of his recent history of blocked coronary arteries; his best option at this time is treatment with a statin drug. Plasmapheresis is reserved for patients with severe hypertriglyceridemia who are at risk for pancreatitis.

A 20-year-old man with no significant past medical history presents with a severe sore throat for the last hour. He reports associated fever, difficulty swallowing, difficulty in opening his mouth, excessive salivation, and a self-described "raspy, harsh voice." He denies chills, sick contacts, otalgia, myalgias, malaise, rashes, wheezing, shortness of breath, and cough. He also denies smoking, drinking, drug use, or recent sexual activity. His physical exam is notable for a toxic-appearing young male patient with tonsillar hyperemia, swelling of the anterior pillar and soft palate, tender cervical lymphadenopathy, and a small left-tonsillar abscess. The primary care provider performs an oropharyngeal culture and refers the patient immediately to the emergency room for further treatment. Question What is the most likely agent isolated from the oropharyngeal culture performed by the provider? 1 Corynebacterium diphtheriae 2 Moraxella catarrhalis 3 Haemophilus influenzae 4 Streptococcus pyogenes 5 Fusobacterium necrophorum

4 Streptococcus pyogenes Explanation The correct response is Streptococcus pyogenes. This patient's presentation is significant for a peritonsillar abscess. Group A hemolytic streptococci (often as part of a mixed flora containing anaerobes) are most commonly isolated. Most common strains are Streptococcus pyogenes (group A streptococcus [GAS]), Streptococcus anginosus, Staphylococcus aureus (including methicillin-resistant S. aureus [MRSA]), and respiratory anaerobes.

A 68-year-old woman presents with shortness of breath, fatigue, dry cough, and swelling in her ankles. Symptoms started around the time of her son's wedding 6 weeks ago. She dismissed them as stress-related, but symptoms have worsened in intensity and frequency. Now she becomes very short of breath with any exertion, such as climbing stairs in her home. She feels like she may pass out and has to sit when she gets lightheaded. Her breathing is more difficult when lying down. She denies productive cough, fever, or chest pain. She has had no medical care for several years. Her past medical history is unremarkable, with no known medical conditions and no surgeries. She is menopausal and does not take any medications, nor does she have any allergies. She denies the use of tobacco, alcohol, and drugs. Vitals are shown in the table. Physical exam was performed by the provider on an earlier shift, and the patient's care was turned over to you. Weight 171 lb Height 65" BMI 28.5 Pulse 101 Blood pressure 158/98 mm Hg Temperature 97.9°F/36.6°C (BNP) Elevated Chest X-ray Perivascular edema, slightly enlarged cardiac shadow 12-lead ekg Mild left ventricular hypertrophy Cardiac echocardiogram Pending Cardiac enzymes Normal Question What physical exam findings would be expected given this patient's history and test results? 1 Absent breath sounds over single lung lobe 2 Dullness to percussion over left upper lung fields 3 Hepatojugular reflux 4 Pericardial friction rub 5 Positive Homan sign

Correct Answer: Hepatojugular reflux Explanation This patient is presenting with an acute exacerbation of heart failure (HF). HF is a common chronic condition and can have acute exacerbations. Common HF presentations include fatigue, dyspnea with exertion, and fluid overload. Hepatojugular reflux (observation of filling of the jugular vein when pushing on the liver of a patient placed in a semi-reclined position), S3 heart sound, peripheral edema, and nonspecific pulmonary findings (such as rales) are often seen in the patient with HF. The cardiac exam may demonstrate tachycardia or even an abnormal rhythm. In this patient with no prior known disorders, several causes could have precipitated her HF.

A 48-year-old man presents to the family practice clinic for evaluation of fatigue, weakness, and nausea. He reports that his symptoms have progressively worsened over the last 6 - 8 months. He reports fatigue despite adequate sleep; he is also experiencing an overall feeling of muscle weakness, nausea with occasional vomiting, a weight loss of about 12 pounds, headaches, and muscle aches. His wife thinks he appears tanned year-round, despite a lack of sun exposure. He admits feeling anxious and somewhat irritable, but he denies any major psychosocial or traumatic events surrounding onset of symptoms. Prior to the onset of his symptoms, he was healthy and active. His past medical history reveals no chronic medical conditions and no medication use; there is no history of surgery; he does not have any allergies. His family history is significant for thyroid disease in a sister and his mother; there is also a history of diabetes mellitus type I in a brother. He teaches high school, and he lives with his wife and children; he denies the use of tobacco, alcohol, and drugs. On physical exam, he is noted to be hypotensive and hyperpigmented. The remainder of his physical exam is normal. Several labs are performed, and the results are as follows: Complete blood count Mildly decreased HB&HT Comprehensive metabolic panel: Mildly decreased sodium and elevated potassium; the rest is normal Adrenocorticotropic hormone (ACTH): Elevated Cortisol (morning level): Decreased ACTH stimulation test: Decreased cortisol Question Once this patient's condition is stabilized and treated with the appropriate medication(s), what should be done to prevent making his condition worse (i.e., health maintenance recommendations)? 1 Avoid any sun exposure 2 Avoid phenylalanine-containing dietary products 3 Increase medication prior to any future surgery 4 Perform yearly renal ultrasounds 5 Switch to salt substitutes

Correct Answer: Increase medication prior to any future surgery This patient is presenting with a corticoadrenal insufficiency, which is also known as Addison's disease. This rare disease is most often linked with autoimmune causes in the United States. The affected patient will have a variety of symptoms related to low glucocorticoid and mineralocorticoid production. Signs and symptoms include fatigue, nausea, vomiting, headaches, anorexia, myalgias, arthralgias, muscle weakness, anxiety, mental irritability, hyperpigmentation, hypotension, hyponatremia, hyperkalemia, anemia, and many more. The mainstay of treatment is steroid replacement, often with oral hydrocortisone. Patients with adrenal insufficiency should be instructed to increase medication prior to any future surgeries, as well as at times of trauma and infection, due to their bodies' inability to naturally increased steroid production during times of physiologic stress.

A 55-year-old man presents with a 12-hour history of severe epigastric abdominal pain that radiates into his back, nausea, vomiting, and chills. His pertinent physical examination findings include oral temperature of 102°F and epigastric tenderness upon palpation. He is admitted to the hospital for management of his condition. Question What criterion would increase the likelihood of a severe attack associated with a high incidence of short- and long-term morbidity and mortality? 1 White blood cell (WBC) count of 10,000 cells/mm3 upon admission 2 A 14% decrease in hematocrit at 48 hours after admission 3 Serum glucose of 130 mg/dL upon admission 4 Aspartate transaminase of 240 U/dL upon admission 5 Serum calcium of 10 mg/dL 48 hours after admission

Correct Answer: A 14% decrease in hematocrit at 48 hours after admission Explanation A 14% decrease in hematocrit at 48 hours after admission is the correct response. A patient who is admitted to the hospital for treatment of acute pancreatitis is assessed using the Ranson criteria to determine the severity of their disease, which in turn helps determine prognosis. About 70-80% of cases of acute pancreatitis are considered mild and result in virtually no morbidity or mortality. The remainder are severe attacks and have a 10-30% mortality rate. Fast identification of the severe cases is helpful to reduce the morbidity and mortality for each patient. There are 11 Ranson criteria, 5 of which are determined upon admission, and 6 at 48 hours after admission. Patients who have 2 or fewer of the criteria have minimal mortality. Patients with 3-5 of the criteria have about a 10-20% chance of mortality. Patients with 5 or more of the criteria have at least a 50% mortality rate. A patient with at least a 10% decrease in their hematocrit at 48 hours after admission meets one of the Ranson criteria, so the correct answer is a 14% decrease in hematocrit 48 hours after admission. WBC count of 10,000 cells/mm3 is not one of the Ranson criteria. A WBC count of >16,000 cells/mm3 would meet one of the Ranson criteria. Serum glucose of 130 mg/dL upon admission is also not one of the Ranson criteria. The patient's serum glucose would have to be at least 200 mg/dL to count as one of the Ranson criteria and increase the severity of their disease. Aspartate transaminase of 240 U/dL upon admission is not one of the Ranson criteria either. The patient's aspartate transaminase would have to be at least 250 U/dL to count as a Ranson criteria and increase the severity of their disease. A serum calcium of 10 mg/dL 48 hours after admission is not one of the Ranson criteria. If the patient's serum calcium is under 8 mg/dL 58 hours after admission, this would count as one of the Ranson criteria.

A 64-year-old man presents with progressive weakening of his urinary stream, nocturia, post void dribbling, and the sensation that he is not emptying his bladder. Annual screening blood work recently revealed a serum prostate-specific antigen of 4.3 ng/mL. Question What is the most important component to your physical examination of this patient? 1 Abdominal examination, including liver percussion 2 Percussion for costovertebral angle tenderness 3 Scrotal examination with and without Valsalva 4 Abdominal examination, including digital rectal exam 5 Scrotal examination and inspection of urethral meatus

Correct Answer: Abdominal examination, including digital rectal exam Explanation The portion of your examination on this patient that would be the most crucial is the abdominal examination, including digital rectal exam (DRE), because a male patient in this age group is at risk for both benign prostatic hyperplasia (BPH) and prostate cancer. The lower urinary tract symptoms listed are indicative of BPH, as is the minimally elevated prostate-specific antigen. It would be important to rule out prostate cancer prior to treatment of the BPH. Prostate cancer screening includes a DRE, which is one reason that abdominal examination, including digital rectal exam (DRE), is the correct answer. A healthcare provider is also able to determine whether a patient's prostate is enlarged by doing a DRE, which is an extension of the abdominal examination. Suprapubic palpation of the bladder during the abdominal examination may indicate a large amount of retained urine secondary to BPH. Percussion of costovertebral angle tenderness (CVAT) is typically used to detect discomfort related to hydronephrosis. Hydronephrosis could be present secondary to obstruction of the urinary tract by various causes. It is true that extreme cases of BPH can cause hydronephrosis and therefore CVAT. This does not, however, make percussion for CVAT a more important component than abdominal examination and DRE. The latter is used for diagnosis, and the former can help diagnose a complication. Scrotal examination should be part of any genitourinary examination of a male patient, but it is not needed to make this patient's diagnosis. The lower urinary tract symptoms described are not symptoms of any scrotal abnormality. They are also not symptoms of any hepatic abnormality, so liver percussion would not be imperative in this case.

A 10-year-old boy presents with a 1-week history of progressing joint pain. The pain started in his ankles, and then progressed to his knees; his hips are now starting to hurt. His ankles feel slightly better. He had contact with someone who had strep throat within the past couple of weeks. The patient's heart rate is 130. On exam, there is erythema and edema over the knees and hips as well as minimal edema over the ankles. A high-pitched holosystolic murmur is noted over the apex and radiates to the axilla with a noted friction rub. Question Based on the above history and physical exam, what is the most likely diagnosis? 1 Juvenile idiopathic arthritis 2 Systemic lupus erythematosus 3 Acute rheumatic fever 4 Kawasaki disease 5 Septic arthritis

Correct Answer: Acute rheumatic fever Explanation Acute rheumatic fever is the correct response; the scenario identifies 2 major Jones criteria: migratory polyarthritis and carditis. Arthritis with acute rheumatic fever typically starts in the legs and then migrates; most commonly, it affects the large joints. The pain can subside within 1 - 2 days, and the arthritis starts affecting another joint. Swelling and redness are common over the joints involved. Typically, a mitral or aortic regurgitation murmur is noted on exam. A friction rub indicates involvement of the myocardium. Polyarthritis and carditis are the most common major Jones criteria for acute rheumatic fever. A confirmed group A Streptococcus test needs to be done, but with the sick contact, you can safely assume the patient has strep. Juvenile idiopathic arthritis is incorrect; it would not explain the tachycardia or other abnormal cardiac physical exam findings. The eyes can be involved with juvenile idiopathic arthritis, which was not the case with the above patient. Systemic lupus erythematosus is incorrect; based on his history, the patient does not have the 4 required criteria from the American College of Rheumatology necessary to make the diagnosis. No rash was noted on exam. Nothing was stated about pleuritis, anemia symptoms, seizures, or positive ANA. Kawasaki disease is incorrect because there was nothing stated about a long-lasting high fever, conjunctivitis, rash, 'strawberry' tongue, redness on palms and soles, and/or swollen lymph nodes. Joint pain can present during the 2nd phase of Kawasaki's disease, but based on the overall scenario, Kawasaki's disease is an unlikely answer.

On routine EKG, a 59-year-old Caucasian man is found to have developed new-onset atrial fibrillation with a ventricular response rate of 98 beats per minute. The atrial fibrillation is asymptomatic; it has not been associated with dizziness, palpitations, chest pain, or shortness of breath. Duration of atrial fibrillation is unknown. His last EKG was 6 months ago. The patient's past medical history is notable for hypertension and coronary artery disease with prior inferior MI and RCA stent. He has no history of stroke or TIA. Allergies: bee stings. Usual home medications: aspirin 81 mg daily, hydrochlorothiazide 25 mg daily, metoprolol succinate (Toprol XL) 25 mg daily. He quit smoking 20 years ago. The patient does not want to take warfarin anticoagulation because of the need for frequent lab testing with warfarin. The patient's dose of beta-blocker is adjusted. Echocardiogram and thyroid function tests are ordered for further evaluation. In addition, a stress echo test is ordered to rule out the progression of CAD. The option of cardioversion and the timing thereof is discussed with the patient. Question What other action is most appropriate for this patient? 1 Advise the patient that he can take aspirin as an alternative to warfarin for prevention of stroke. 2 Advise the patient that he is at low risk of stroke and does not require anticoagulation for prevention of stroke. 3 Advise the patient that his risk of stroke is increased, and recommend that he take a novel anticoagulant for stroke prevention. 4 Advise the patient that if chooses to have immediate transesophageal echocardiogram (TEE) and cardioversion, he will not require anticoagulation. 5 Advise the patient to take clopidogrel (Plavix) 75 mg daily for prevention of stroke.

Correct Answer: Advise the patient that his risk of stroke is increased, and recommend that he take a novel anticoagulant for stroke prevention. Explanation The correct answer is to advise the patient that his risk of stroke is increased, and recommend that he take a novel anticoagulant for stroke prevention. The risk of stroke is increased 5 times with nonvalvular atrial fibrillation; stroke associated with atrial fibrillation is more likely to be fatal or have a worse outcome. Currently, the CHA2DS2-VASC score is recommended for use as a screening tool for stroke risk in the setting of atrial fibrillation: Congestive Heart Failure 1 point if yes Hypertension 1 point if yes Age 1 point if 65-74; 2 points if ≥75 Diabetes 1 point if yes Stroke, TIA, or thromboembolism history 2 points if yes Sex 1 point if female Vascular Disease* 1 point if yes Maximum Point Value 9 points maximum *defined as previous MI, peripheral arterial disease, or aortic plaque A total score of 0 is low-risk; either no anticoagulation or aspirin alone is recommended. A score of 1 is intermediate risk; either aspirin or oral anticoagulation is recommended. A score of 2 or higher is high-risk; oral anticoagulation is recommended. This patient has hypertension and vascular disease with prior MI. His CHA2DS2-VASC score is 2; oral anticoagulation is recommended. The choice of anticoagulant agent should be based on shared decision making with the patient, including risks for both stroke and bleeding and the patient's preferences. Options for anticoagulation, according to ACC/AHA/HRS 2014 guidelines, include warfarin (Coumadin), dabigatran (Pradaxa), rivaroxaban (Xarelto), or apixaban (Eliquis). Because the patient's CHA2DS2-VASC score of 2 is high-risk, it is incorrect to advise the patient that he can take aspirin as an alternative to warfarin for prevention of stroke OR to advise the patient that he is at low risk of stroke and does not require anticoagulation for prevention of stroke. If the atrial fibrillation duration was known to be 48 hours or less, then TEE followed by cardioversion without anticoagulation would be an option; however, the duration of atrial fibrillation is unknown and so requires anticoagulation regardless of TEE/cardioversion. It is incorrect to advise the patient that if he chooses to have immediate TEE and cardioversion, he will not require anticoagulation. Clopidogrel has been evaluated for stroke prophylaxis in combination with aspirin, but it was associated with an increased major bleeding risk and was inferior to warfarin for high-risk patients. It is incorrect to advise the patient to take clopidogrel (Plavix) 75 mg daily for prevention of stroke.

An 8-year-old girl presents with her mother to the pediatrician's office with persistent clear nasal drainage and nighttime cough for the past month. Her physical examination reveals clear rhinorrhea, dark circles under her eyes, and a transverse nasal crease. What is the most likely diagnosis? 1 Nasal foreign body 2 Sinusitis 3 Allergic rhinitis 4 Upper respiratory infection 5 Influenza

Correct Answer: Allergic rhinitis Explanation Allergic rhinitis is a disorder of the nasal mucosa caused by IgE-mediated inflammation. It can present at almost any age, but incidence decreases with advanced age. In children, allergic rhinitis can occur along with atopic dermatitis and/or food allergies. The symptoms typically include persistent clear nasal drainage, stuffiness, and/or congestion. Pruritus of the eyes, nose, palate, or ears can occur. Patients often sneeze and have postnasal drainage that can lead to a bothersome cough, particularly at night. The physical examination can reveal allergic shiners, which are dark circles under the eyes. Presence of a clear nasal discharge is usually seen as well. A transverse nasal crease can be seen in children from them rubbing their noses upwards rather than using a tissue. A nasal foreign body is usually unilateral, so the drainage is also usually unilateral. The drainage would not usually be clear either, but purulent with an odor. Sinusitis would present more acutely than with a month of symptoms, and any drainage is likely to be thick and purulent. Physical examination would not likely reveal allergic shiners or a crease on the nose, but it may reveal fever, edema of the nasal mucosa, and tenderness over the sinuses. The symptoms of allergic rhinitis and upper respiratory infection (URI) may be very similar, but patients with a URI would likely not be symptomatic for a month and might have various systemic symptoms. A nasal crease and allergic shiners are also not likely to be present.

A 24-year-old man was playing basketball when he jumped and landed on another player's leg. The patient grabbed his left ankle immediately and screamed in pain. His whole ankle started to swell, and he had difficulty bearing weight. His friends took him to the emergency room. On exam, the patient had tenderness to the tip of the lateral malleolus. Pain is elicited with forced ankle inversion. Talar tilt test and anterior drawer test cannot be assessed due to swelling and pain. X-rays are negative for a fracture. Question What ligament did the patient most likely sprain? 1 Anterior tibiotalar ligament 2 Tibiocalcaneal ligament 3 Posterior tibiotalar ligament 4 Tibionavicular ligament 5 Anterior talofibular ligamen

Correct Answer: Anterior talofibular ligament Explanation The anterior talofibular ligament is the only ligament listed above that is in the lateral aspect of the ankle. The mechanism of injury and physical exam points to the lateral part of the ankle. The other choices compose what is commonly known as the deltoid ligament complex and are all found medially. The patient jumped and most likely had his foot in a plantar flexion; upon landing on another player, the ankle was inverted. The anterior talofibular ligament is the first to be injured and can be the only ligament injured in lateral ankle sprains. The stronger force, however, can injure the other 2 ligaments in the lateral aspect of the ankle: the calcaneofibular ligament and posterior talofibular ligament. The special physical exam tests have very limited usefulness in this case since this is an acute injury; they should be used after the pain and swelling have gone down.

A 30-year-old male presents for routine follow-up. He has a history of intermittent headaches, low back pain, knee pain, and dysuria over the last year. Additionally, he notes nausea, diarrhea, poor libido, and extremity numbness. He states he feels worthless. He admits to daily alcohol use in an attempt to feel better. Lab studies, urine testing, plain radiographs, CT scans, and MRIs are all unremarkable and show no explanation for his symptoms, but he is consumed by worry about his illness. Question What is correct regarding this patient's condition? 1 Typical onset occurs in the elderly with underlying vascular disease. 2 Associated personality disorders and depression are common. 3 The cornerstone of treatment is benzodiazepines. 4 Highly-educated married Caucasian men are the most likely to be affected. 5 Serological antibody testing is the most specific diagnostic test available.

Correct Answer: Associated personality disorders and depression are common. Explanation The correct response is that associated personality disorders and depression are common. This patient's most likely diagnosis is somatic symptom disorder. Patients with somatic symptom disorder have maladaptive thoughts, feelings, and behaviors that define the disorder, in addition to their somatic symptoms (e.g., depression, anxiety, suicidal gestures, substance abuse). They may be addicted to prescribed medications; at times they may exhibit drug-seeking behaviors. Symptomatic presentation, which can be quite dramatic, is frequently associated with concurrent psychosocial stressors. There is no specific psychopharmacologic treatment for somatic symptom disorder. These patients frequently suffer from comorbid psychiatric disorders (e.g., panic disorder or depression), however, which should be appropriately treated. Patients with somatic symptom disorder are at risk for iatrogenic complications of invasive or therapeutic procedures. Habituation to prescribed analgesics or anxiolytics also occurs frequently. Clinicians must exercise caution when prescribing any potentially lethal medication for these patients because they are prone to impulsive acting-out behaviors, including suicide attempts. Individuals who meet the full criteria for somatic symptom disorder tend to be female, unmarried, non-Caucasian, poorly educated, and from rural areas. There are no specific laboratory findings for somatic symptom disorder; the diagnosis is based on a lack of objective evidence to substantiate physical disease.

A 32-year-old woman is 2 hours status post-cesarean delivery of a twin gestation at 36 weeks. Her pregnancy was uncomplicated. She presented in early labor, which became prolonged despite oxytocin infusion. A cesarean section was performed when her labor became non-progressive. In the recovery area, she notes nausea and lightheadedness. On exam, her heart rate is 133 beats per minute and blood pressure is 76/42 mm Hg. Significant vaginal bleeding is noted; abdominal palpation reveals a soft uterus. Question What intervention would be most appropriate? 1 Administration of intravenous magnesium 2 Bimanual uterine massage 3 Immediate surgical exploration 4 Transfusion with fresh frozen plasma 5 Transvaginal ultrasound

Correct Answer: Bimanual uterine massage Explanation The most likely cause of bleeding in this patient is uterine atony. Uterine atony occurs when the uterine myometrium fails to contract following delivery. Contractions of the uterine muscles after delivery normally tamponade bleeding from uterine arterioles. Absence of this response causes continued bleeding, which usually becomes evident soon after delivery. Bimanual uterine palpation revealing a soft "boggy" uterus confirms the diagnosis. Initial treatment involves bimanual uterine massage, which helps promote uterine contractions. Uterotonic agents are also administered. The first-line of therapy is intravenous oxytocin. Second-line therapies, including ergot alkaloid derivates and prostaglandins (e.g., Hemabate), are used when oxytocin therapy is unsuccessful. When these measures are unsuccessful, surgical interventions are required, the most common of which is bilateral uterine artery ligation. Uterine atony can also occur as a result of retained products of conception; these products inhibit uterine contraction. In this case, manual or surgical extraction is necessary. Intravenous magnesium is a uterine relaxant used to inhibit uterine contractions in settings of premature labor. It has the opposite effect to that desired to treat uterine atony. Transfusion with fresh frozen plasma may be indicated as a temporizing measure in the setting of disseminated intravascular coagulation accompanied by severe bleeding or massive uncontrolled hemorrhage from uterine rupture, neither of which is suspected in this clinical scenario. Transvaginal ultrasound is rarely needed to diagnose postpartum hemorrhage, and it is not the most appropriate next step in light of the patient's condition.

A 4-year-old boy presents with erratic movements of his left arm and leg. These movements have been present since birth and occur on the left side. They have not been getting worse. The boy's mother admits that he has been using his right hand for almost all activities since the age of 1. Further questioning reveals that the child exhibited delayed milestones, such as rolling over at 6 months, sitting up at 1 year, and walking at 2 years. He does not speak, and he does not respond to his mother's verbal commands. His physical exam is noteworthy for spasticity, hyperreflexia, ataxia, involuntary movements, and weakness of his left arm and leg. His left arm and left leg are measured to be shorter and are atrophied in comparison to the right. Additionally, the child does not respond to verbal stimuli. Question What is the most likely diagnosis? 1 Duchenne muscular dystrophy 2 Lesch-Nyhan syndrome 3 Cerebral palsy 4 Huntington's disease 5 Rett syndrome

Correct Answer: Cerebral palsy The most likely diagnosis in this patient is cerebral palsy. Cerebral palsy is a chronic, non-progressive impairment of muscle tone, strength, coordination, or movements. Cerebral palsy is due to cerebral insult or injury before birth, during delivery, or in the perinatal period. The most common form of cerebral palsy (75% of cases) involves spasticity of the limbs. Depending on the type and severity of the motor deficits, associated neurologic deficits or disorders may occur as follows: seizures in up to 50%; mild intellectual disability in 26%; and severe intellectual disability in up to 27%. Disorders of language, speech, vision, hearing, and sensory perception are found in varying degrees and combinations. Gross motor milestones of concern with cerebral palsy include head control at age 2 months, rolling at age 4 months, sitting at age 6 months, and walking at age 1 year. Infants with cerebral palsy may have significantly delayed gross motor milestones or show an early hand preference when younger than 1.5 years, suggesting the relative weakness of 1 side. While this patient demonstrates a hemiplegic pattern, other patterns are possible, such as monoplegia or quadriplegia.

A 35-year-old woman presents with a 24-hour history of fever, right flank pain, vomiting, dysuria, and hematuria. A urinalysis reveals large numbers of red and white blood cells, as well as leukocyte esterase, and a subsequent urine culture was positive for >100,000 Escherichia coli. The patient denies having a history of renal calculi, and a bedside renal ultrasound does not show any stones or hydronephrosis bilaterally. She is diagnosed with a right-sided pyelonephritis and admitted for pain control, hydration, and IV antibiotic therapy. Her symptoms, including her fever, abate quickly, and she is discharged after 48 hours of being afebrile. Question Which prescription would she most likely receive upon discharge? 1 Motrin 800 mg PO q 8 hours prn pain 2 Nitrofurantoin 100 mg PO BID x 7 days 3 Ciprofloxacin 500 mg PO BID x 14 days 4 Fluconazole 150mg PO daily x 7days 5 Amoxicillin 500 mg PO BID x 14 days

Correct Answer: Ciprofloxacin 500 mg PO BID x 14 days Explanation Ciprofloxacin 500 mg PO BID x 14 days is the correct answer. Patients with pyelonephritis who are sick enough to be treated as an inpatient receive IV antibiotics until they have been afebrile for 24-48 hours. They also must be able to tolerate oral hydration and oral medications before being discharged. Upon discharge, they will be given a prescription for antibiotics that will complete at least 2 weeks of antibiotic treatment. Ciprofloxacin has good coverage for E. coli urinary tract infections, and given at 500 mg PO BID x 14 days would be an appropriate choice as long as sensitivities from her culture showed ciprofloxacin to have sensitivity. Motrin 800 mg PO q 8 hours prn pain is not the correct answer. While Motrin is a good choice for patients who may still have some discomfort related to the pyelonephritis, it is not the most likely prescription for this patient to receive. She had become asymptomatic prior to discharge, so there would not be any reason to prescribe anything to help with pain or discomfort at that time. In addition, she is more likely to receive treatment for her infection than for pain. Nitrofurantoin 100 mg PO BID x 7 days is not the correct answer for several reasons. While nitrofurantoin is often an appropriate antibiotic for E. coli urinary tract infections, it does not achieve tissue levels reliable enough for pyelonephritis treatment. In addition, only 7 days of treatment does not add up to at least 2 weeks total of antibiotics. Fluconazole 150mg PO daily x 7days is not the correct choice. This is an antifungal medication is would not have an indication to use as treatment is this patient case. Amoxicillin 500 mg PO BID x 14 days is not the correct answer. While 14 days of antibiotics is a good length of time, amoxicillin does not have good coverage for E. coli, so it would not be a good choice to treat an E. coli-related pyelonephritis.

A 35-year-old woman with no significant past medical history presents with a feeling of constant worry for the past year. She states that she is anxious about numerous elements in her life (e.g., constant deadlines at her job, obligations to her family, financial constraints, repairs to her car, the health of her 2 young children). She denies any cigarette, alcohol, drug use, or any history of abuse. She also reports fatigue, restlessness, and muscle tightness, especially around her neck. She denies any chest pain, shortness of breath, palpitations, cough, edema, diaphoresis, or changes in weight, diet, or caffeine intake. Her physical exam reveals cervical myospasms. Otherwise, she has normal vital signs, a non-tender and non-enlarged thyroid gland, a normal skin exam, and a normal cardiovascular exam. Question What medication is expected to induce the most rapid clinical response in this patient? 1 Escitalopram 2 Oxazepam 3 Lithium 4 Buspirone 5 Diazepam

Correct Answer: Diazepam Explanation This patient demonstrates manifestations consistent with generalized anxiety disorder (GAD). A combination of pharmacologic and psychotherapeutic interventions is most effective in GAD. Diazepam is a benzodiazepine that is lipid-soluble, is absorbed rapidly, and has a rapid onset of action. Certain SSRIs (e.g., escitalopram) are effective, but typically only after being taken for at least a few weeks. Unlike diazepam, oxazepam is a benzodiazepine that has a slow onset of action. Buspirone is also effective, but it too can take at least 2 weeks before it begins to help. Lithium is the mainstay of treatment in bipolar disorder, and as prophylaxis in recurrent mania and depression. The therapeutic effects of lithium may not appear until 7-10 days of treatment.

A 43-year-old woman who works as a secretary and spends most of her day typing on the computer has symptoms of carpal tunnel syndrome. What diagnostic test is most useful in confirming your diagnosis? 1 Electromyogram 2 Computerized tomogram (CT) of the wrist 3 Magnetic resonance image (MRI) of the wrist 4 Plain radiographs of the wrist 5 Erythrocyte sedimentation rate (ESR)

Correct Answer: Electromyogram Explanation Electrophysiologic (electromyogram) testing is the most useful confirmatory test for carpal tunnel syndrome (CTS), but these tests must be used with caution. Some patients have no clinical signs or symptoms but will have an abnormal EMG; as many as 5-10% of patients with CTS will have normal results. In general, the diagnosis of CTS is based on your H&P, with EMG use to confirm your suspicion. A positive Phalen's and Tinel's sign would demonstrate paresthesia in a median nerve distribution, a positive Tinel's sign via percussion of the median nerve at the wrist, and positive Phalen's after passive flexion of the hand at the wrist. They are used to help in diagnosis but are not very sensitive or specific tests. Electrodiagnostic studies are often needed to confirm carpal tunnel syndrome. CT is used more often for boney pathology of the wrist. MRI may show swelling within the carpal tunnel but is not diagnostic. Plain films should be taken if there is limited range of motion of the wrist but is not diagnostic of CTS. ESR may be slightly elevated with CTS but is not specific for it.

A 35-year-old woman presents with a 24-hour history of purulent drainage and erythema of her right eye. After a brief physical examination, cultures of the drainage are taken and she is started on a medication prophylactically that would cover the most common bacterial causes of conjunctivitis (including sexually transmitted diseases). On what medication (in either an oral or topical form) would she most likely be started? 1 Erythromycin 2 Tetracycline 3 Bacitracin 4 Olopatadine 5 Acyclovir

Correct Answer: Erythromycin Explanation This patient is exhibiting symptoms that suggest disorganized schizophrenia. Disorganized schizophrenia is characterized by social behavior that is out of context, with marked incoherence and an incongruous or silly affect. The patient's appearance may be bizarre, and motor activity may escalate to frenzied excitement. Inappropriate behavior including stereotypy, echopraxia, and disturbed interpersonal relationships may occur. Verbal changes such as echolalia, neologisms, concrete language, clanging, incoherent speech, and verbigeration are common. When the patient's behavior shows gross disorganization, hospitalization is often necessary, especially when the behavior is coupled with evidence or threat of harm to self or others.

A 22-year-old nulligravid Caucasian woman presents to your OB/GYN clinic for evaluation of menstrual changes. She reports having normal menses since menarche at age 13, but then noticed she has only had 3 menses over the last year, with the last one about 3 months ago. She denies dysmenorrhea and menorrhagia, any major life event changes, and any major stressors. Review of systems is positive for a recent increase in severe headaches. Past medical and surgical histories are unremarkable, with no known medical conditions or surgeries. She takes no chronic medications but has been using over-the-counter analgesics for her headaches. She has no allergies. Social history reveals the patient has never been sexually active and lives at home with her parents and siblings. She does not use alcohol, tobacco, or drugs. Vitals and a urine specimen for hCG are obtained from your medical assistant prior to physical exam. Weight 152 lb Height 67" Pulse 70 Blood pressure 112/66 Temperature 97.2°F Urine hCG Negative Question What physical exam finding would be seen in a condition that explains both the headaches and amenorrhea in this patient? 1 Acanthosis nigricans 2 Galactorrhea 3 Hirsutism 4 Imperforate hymen 5 Thyroid nodule

Correct Answer: Galactorrhea Explanation Galactorrhea would suggest elevated levels of serum prolactin and a prolactinoma in the pituitary as the underlying cause. A prolactinoma is associated with menstrual changes (often oligomenorrhea and amenorrhea), galactorrhea, headaches, and visual changes. Acanthosis nigricans is a hyperpigmentation of the skin, especially in the neck folds, and is associated with elevated insulin levels, diabetes, and hyperandrogenism. These conditions are commonly associated with polycystic ovarian syndrome and amenorrhea or oligomenorrhea, but there is no direct association with headaches. Hirsutism is a sign of hyperandrogenism and is often seen in women with polycystic ovarian syndrome, although it can occur due to other causes. The serum testosterone levels may or may not be elevated. Hyperandrogenism is linked with menstrual changes but not directly with headaches. Imperforate hymen is a suspected cause in primary amenorrhea, in which the woman has never had a menstrual cycle. This woman presents with secondary amenorrhea, so her hymen would not be expected to be intact. This condition also has no influence on headaches. Thyroid nodules and overall thyroid dysfunction can play a role in menstrual regularity, but a nodule would not directly cause headaches.

A 41-year-old man presents for a physical. He has not had a routine physical in a couple of years. Upon taking a review of systems, you become suspicious that the patient is not present for routine health maintenance alone. The patient admits to various abnormal symptoms that result in the patient receiving an order slip for a 24-hour urine study looking for vanillylmandelic acid (VMA), catecholamines, and metanephrines. Question What triad of symptoms indicates a need for this particular 24-hour urine study? 1 Dry skin, fatigue, and shortness of breath 2 Fatigue, flushing, and constipation 3 Headache, dry skin, and nausea 4 Flushing, constipation, and nausea 5 Headache, palpitations, and diaphoresis

Correct Answer: Headache, palpitations, and diaphoresis Explanation Headache, palpitations, and diaphoresis together make up the "classic triad" in patients with a pheochromocytoma. A pheochromocytoma is a catecholamine-producing tumor that is most commonly found in the adrenal gland superior to the kidney. It is commonly diagnosed due to hypertension, but it causes less than 1% of hypertension cases. The symptom triad listed above has been found to have a specificity of almost 94% and a sensitivity of almost 91% for patients with pheochromocytomas. Patients may also experience other symptoms, including constipation, weight loss, flushing, and dizziness. Since the tumor produces catecholamines, excess production is usually measured by determining the levels of excreted metabolites during a 24-hour period by a 24-hour urine. Abdominal MRI can then be used to confirm the presence of the adrenal tumor.

A 56-year-old man with a past medical history of type 2 diabetes mellitus, hypertension, myocardial infarction 1 year ago, and a 50 pack-year smoking history presents with unilateral leg edema and leg pain for the last 6 hours. His physical exam is remarkable for an elevated systolic blood pressure, obesity, and localized left lower extremity edema, erythema, and tenderness to the left lateral calf. Question What additional physical exam finding would be most consistent with a diagnosis of superficial thrombophlebitis in this patient? 1 Linear induration along the course of a vein 2 Peripheral cyanosis 3 A positive Homan's sign 4 Reduced amplitudes of the dorsalis pedis and tibialis anterior pulses 5 A shallow well-circumscribed ulcer above the medial malleolus

Correct Answer: Linear induration along the course of a vein Explanation Superficial thrombophlebitis is a common inflammatory-thrombotic disorder in which a thrombus develops in a vein located near the surface of the skin. Superficial thrombophlebitis usually presents with pain, swelling, redness, and tenderness of superficial veins. When superficial thrombophlebitis occurs in the short or long saphenous veins, there is usually redness, tenderness, and often linear induration that follows the course of the involved vein (medial calf or thigh). Peripheral cyanosis and reduced peripheral pulse amplitudes are examination findings common to peripheral arterial occlusive disease. Homan's sign (calf pain precipitated by dorsal flexion of the foot) is present in fewer than 50% of deep venous thrombosis cases. Physical examination of chronic venous insufficiency shows orange-brownish skin discoloration at the level of the ankle with hemosiderin deposition, lower extremity edema, superficial varicosities, and ulceration; ulcers usually occur at the medial and lateral malleoli of the ankle.

A 37-year-old woman was diagnosed as HIV-positive 3 years ago; she is unable to work and is physically debilitated, so she asks you to provide her with medications with which to take her own life. Question What is the most common emotional disorder associated with such a request? 1 Bipolar I disorder, manic type 2 Borderline personality disorder 3 Factitious disorder 4 Major depressive disorder 5 Schizophrenic disorder

Correct Answer: Major depressive disorder explanation Many people who request assisted suicide have 1 of 2 conditions present: either a poorly controlled painful condition or severe depression. If the painful condition is adequately treated or the depression is brought under good medical control, the request for assistance in terminating the situation is typically withdrawn. It is important to note that bringing these conditions under control requires the intervention of caregivers who are specifically trained in the management of these 2 conditions; primary care providers usually are not adequately trained to address these difficult presentations. While patients who are diagnosed with bipolar disorder, borderline personality disorder, and schizophrenic disorder often attempt suicide (and frequently complete those attempts), they do not generally ask their physician for assistance in the suicide. Persons with factitious disorder are seeking primary gain, often for dependency needs, and are seeking to enter the "sick role" and not the "dead role."

A 74-year-old man presents after his wife witnessed him grab his head in pain and fall to the floor. He has not regained consciousness. His current blood pressure is 150/96 mm Hg and his heart rate is 65 bpm. Emergent head CT shows a subarachnoid hemorrhage. Question In addition to life-saving interventions, what prescription medication will most benefit this patient at this time? 1 Furosemide (loop diuretic) 2 Prednisone (glucocorticoid) 3 Tranexamic acid (antifibrinolytic agent) 4 Labetalol (beta blocker) 5 Nimodipine (calcium channel blocker)

Correct Answer: Nimodipine (calcium channel blocker) Explanation Nimodipine (a calcium channel blocker) has been shown to improve outcomes in patients following aneurysmal SAH. The mechanism of action is thought to be prevention of ischemia. Glucocorticoids (e.g., prednisone) are often utilized in patients with SAH because they offer symptomatic relief of headache and neck pain, but they have not been proven to decrease cerebral edema. Antifibrinolytic agents (e.g., tranexamic acid) can be utilized in patients with a diagnosed aneurysm who cannot undergo directed treatment, but they are not routinely used following aneurysmal rupture. Labetalol (a beta blocker) may be utilized to treat elevated BP, but it must be used with caution because it can also decrease cerebral perfusion. Diuretics (e.g., furosemide) have no identified role in the treatment of aneurysmal SAH.

A 57-year-old woman is undergoing a workup by her primary care provider for abdominal pain. The pain is in her left lower quadrant, intermittent, "crampy," and has been present for about 2 months. As an initial imaging study, she underwent an abdominal ultrasound. The ultrasound was unremarkable except for the presence of a few small (<1cm) stones in her gallbladder. She is currently following up to discuss the results of the ultrasound. Question What treatment is likely to be suggested for the gallstones? 1 No treatment 2 Prophylactic elective cholecystectomy 3 Oral analgesics 4 Urgent cholecystectomy 5 Oral dissolution therapy

Correct Answer: No treatment Explanation Gallstones on the patient's ultrasound is an incidental finding and not the cause of the patient's pain. As such, further investigation of the cause of the pain is warranted, and the correct answer is no treatment. If the gallstones were causing symptoms, this would be known as cholecystitis and the patient would have different abdominal complaints. Only 5-10% of patients with cholelithiasis actually have cholecystitis, and the symptoms could include right upper quadrant abdominal pain, epigastric abdominal pain, nausea, vomiting, indigestion, or fatty food intolerance. Since this patient is not experiencing symptoms related to the stones, the stones are under 2-3 cm, and the gallbladder is not reported as "calcified" in general, she does not need to undergo treatment for the incidental finding of cholelithiasis. Prophylactic elective cholecystectomy is incorrect. Treatment is not needed for asymptomatic stones. If her stones were greater than 2-3 cm, if her gallbladder were calcified (risk for gallbladder cancer), or if she were suffering from recurrent pancreatitis secondary to microlithiasis, then a prophylactic cholecystectomy would be indicated. Oral analgesics is incorrect. The patient has noted pain, but the source has not yet been determined. Whether the patient receives oral analgesics for the left lower quadrant pain while undergoing further workup would be up to the provider. However, analgesics would not be warranted for the cholelithiasis, as the stones are asymptomatic. Urgent cholecystectomy is typically performed for patients who have severe symptoms, particularly pain that is intolerable or severe vomiting. Cholecystectomy is usually performed electively (for reasons discussed in "prophylactic elective cholecystectomy") and very rarely needs to be done on an urgent basis. Oral dissolution therapy is very rarely used today and was much more popular 10-20 years ago. Oral dissolution therapy consists of taking bile acid in pill form to try to dissolve stones. It can work on a small percentage of patients with small cholesterol gallstones, but it has significant negative aspects that have taken the therapy out of popular use. It can be very expensive, can take several years, and has a high recurrence rate. As this patient's stones are asymptomatic, the best choice is to watch the stones conservatively.

A 70-year-old African American man presents to his primary care provider with a complaint of gradual but progressive dyspnea and fatigue on exertion. He notes difficulty in climbing stairs with associated lightheadedness, increased abdominal girth, and swollen bilateral lower extremities. He admits to chronic alcohol use but denies any heart disease, chest pain or pressure, diaphoresis, palpitations, a history of diabetes, cigarette smoking, or claudication. His physical exam reveals sinus tachycardia, bibasilar rales, a laterally-displaced PMI, an elevated JVP, an S3 gallop, a mitral regurgitation murmur, peripheral edema, and abdominal ascites. Question What is the next most appropriate step in the management of this patient? 1 Obtain an echocardiogram. 2 Refer the patient for cardiac catheterization. 3 Perform a cardiac biopsy. 4 Order coxsackie and adenovirus serological titers. 5 Order cardiac troponin levels.

Correct Answer: Obtain an echocardiogram. This patient has symptoms and signs that suggest heart failure. Echocardiography is done to look for signs consistent with heart failure and potential causes of heart failure. In this case, patient is found to have dilated cardiomyopathy. Chronic alcohol abuse is a major risk factor for the development of congestive cardiomyopathy, accounting for up to 45% of all dilated cardiomyopathies. After the diagnosis of dilated cardiomyopathy is confirmed by echocardiography, further evaluation is directed to ascertain whether myocarditis is present. An echocardiogram is indicated to exclude unsuspected valvular or other lesions and confirm the presence of dilated cardiomyopathy and the reduced systolic function (as opposed to pure diastolic heart failure). Four-chamber dilatation is a common but not uniform finding. Other findings include increased left ventricle end-diastolic diameter and volume with decreased fractional shortening, thinning left ventricle walls, increased E point-septal separation, left atrial enlargement, and limited mitral and aortic valve opening caused by low stroke volume. Intracardiac thrombi may also be observed. Cardiac catheterization is seldom of specific value unless myocardial ischemia or left ventricular aneurysm is suspected. Myocardial biopsy is rarely useful in establishing the diagnosis, though occasionally the underlying cause (e.g., sarcoidosis, hemochromatosis) can be discerned. Biopsy is most useful in transplant rejection.

A 53-year-old man with a 40 pack-year smoking history presents with a 10-month history of an intermittent cough with productive sputum. He admits to progressive exertional shortness of breath, which recently has limited his activity to climbing 1 flight of stairs or walking 3 city blocks. He denies diaphoresis, fever, chills, chest pain, palpitations, audible wheezing, pleurisy, peripheral edema, hemoptysis, abdominal pain, reflux, regurgitation, diarrhea, melena, or hematochezia. He also denies travel, sick contacts, and drug or alcohol use. His general survey reveals an overweight male with an odor of smoke and nicotine staining of his fingernails. His nails also demonstrate digital clubbing. His pulmonary exam reveals a prolonged expiratory phase, barrel chest, poor diaphragmatic excursion, and wheezing to auscultation. Pulmonary function testing shows airflow obstruction with a reduction in FEV1 and FEV1/FVC ratio; increases in total lung capacity, functional residual capacity, and residual volume were noted. Question What intervention has been demonstrated to influence the natural history of this patient's illness? 1 Intravenous diuretics 2 Inhaled anticholinergic agents 3 Smoking cessation 4 Long-acting inhaled agonists and glucocorticoids 5 Antibiotic prophylaxis

Correct Answer: Smoking cessation Explanation This patient's diagnosis is most consistent with chronic obstructive pulmonary disease. Only 3 interventions: smoking cessation, oxygen therapy in chronically hypoxemic patients, and lung volume reduction surgery in selected patients with emphysema have been demonstrated to influence the natural history of patients with COPD. All other current therapies are directed at improving symptoms and decreasing the frequency and severity of exacerbations. Diuretics are not indicated in the management of COPD. Long-acting inhaled -agonists (e.g., salmeterol) and anticholinergic agents, such as tiotropium or ipratropium bromide, have been shown to improve symptoms and reduce exacerbations; however, it has not been demonstrated that either influences the rate of decline in FEV1. Despite the frequent implication of bacterial infection in COPD exacerbations, chronic suppressive, or "rotating" antibiotics, are not beneficial in patients with COPD.

A 34-year-old man presents for evaluation of 10/10 left calf pain after being run over by a car. Physical exam reveals tense calf swelling, increased pain with passive muscle stretching, and tenderness on calf palpation. Question What other findings are expected in this patient? 1 Pain out of proportion to exam 2 Normal calf compartment pressure 3 Erythematous streaks 4 A palpable calf cord 5 Groin lymphadenopathy

Correct Answer: Pain out of proportion to exam Explanation This patient presents with acute compartment syndrome after a crush injury. Compartment syndrome occurs when excessive muscle swelling increases the pressure within the fascial sheath compartment, compressing vessels and nerves. Compression of the artery leads to ischemia. Compartment syndrome is a medical emergency and can result in the loss of the limb if the pressure is not relieved immediately. Compartment syndrome is a clinical diagnosis based on the 6 Ps (pain out of proportion, pressure on palpation, paresthesia, paralysis, pallor, and pulselessness). In addition to the 6 Ps on PE, diagnosis is confirmed by obtaining the compartment pressure, in which anything above 10 mm Hg is abnormal. At 20 mm Hg, ischemia can occur, and anything above over 30 mm Hg requires an emergent fasciotomy. Streaking erythema is a sign of lymphangitis, which is inflammation or infection that has spread to the lymphatic system. A palpable calf cord can be appreciated in patients who present with a DVT or varicose veins. Lymphadenopathy is swelling of a lymph node secondary to infection, autoimmune disease, or malignancy.

A 59-year-old woman presents for a routine annual physical. She is a housewife who does not smoke or drink, and her only significant past medical history is for an appendectomy and radiation treatment for acne as a child. She is postmenopausal, but she uses natural supplements rather than hormone replacement. She is not on any prescription medications. She completed some routine lab work prior to the office visit. The results are entirely within normal limits, including cholesterol panel, glucose, TSH, and T4. Her only complaints are recent dysphagia and sensation of fullness in her neck. Physical examination reveals a large nodule on the right lobe of patient's thyroid. The patient is referred to an endocrinologist and undergoes an ultrasound of the thyroid, which is followed by a thyroid scan. The results confirm a 1.7 cm solid nodule on ultrasound that is deemed "cold" on thyroid scan. Question Prior to scheduling a fine needle aspiration and based on the information you have, what is the most likely diagnosis? 1 Hashimoto's thyroiditis 2 Medullary thyroid carcinoma 3 Parathyroid cyst 4 Graves' disease 5 Papillary thyroid carcinoma

Correct Answer: Papillary thyroid carcinoma Explanation The correct answer is papillary thyroid carcinoma. The patient has 3 risk factors for thyroid cancer, including female sex, age over 40, and history of radiation to face/neck region for acne. Thyroid function tests are typically normal in patients with thyroid cancers. The diagnostic findings of a solid cold nodule are also suspicious for a thyroid cancer. Those findings can, however, be seen in benign nodules as well. Given the fact that papillary thyroid carcinoma is the most common type of thyroid cancer (about 60% of all thyroid cancers) and the patient has a history of radiation to face/neck region, the most likely type of thyroid cancer in this case is papillary. While thyroid nodules can be present in cases of Hashimoto's thyroiditis, they are usually "multinodular" rather than a solitary thyroid nodule. Patients would also usually have more complaints upon presentation, and the physical examination is likely to be normal or reveal a goiter with or without multiple nodules. The thyroid function testing would not be normal. Most commonly, the TSH would be elevated in the face of decreased T4. Medullary thyroid carcinoma is not the most likely diagnosis in this case. The patient is at risk for thyroid cancer, but her history of radiation to face/neck region and the fact that papillary carcinoma is much more common (60% as opposed to 5% of all thyroid cancers) make her type of cancer more likely to be papillary rather than medullary. A parathyroid cyst falls under the category of a benign cyst of the thyroid. Thyroid cysts make up only 15-25% of thyroid nodules, and benign cysts make up 85% of that group. A parathyroid cyst is one of the many types of benign thyroid cysts, and it would not be solid on an ultrasound. Patients will typically have more complaints and positive symptoms when they have Graves' disease, and the examination of their thyroid is not likely to have a solitary thyroid nodule. The thyroid function tests would not be normal and are most likely to show a decreased TSH and elevated T4.

A preterm female infant born to a 32-year-old woman with no known past medical illnesses presents for the infant's 1-week follow-up. The mother reports that the patient is behaving normally and is feeding well. The physical exam is remarkable for a murmur, which is located at the second left intercostal space. The murmur is continuous throughout cardiac systole; it is diastolic, non-radiating, and of a "machinery" quality. There is also a widened pulse pressure. The skin and mucosa are without cyanosis, and there is no evidence of fluid retention. Question Based on the physical exam finding, what is the most likely diagnosis? 1 Atrial septal defect 2 Ventricular septal defect 3 Tetralogy of Fallot 4 Patent ductus arteriosus 5 Pulmonary stenosis

Correct Answer: Patent ductus arteriosus Explanation This infant is most likely exhibiting signs consistent with patent ductus arteriosus. Patent ductus occurs more commonly in females and has an incidence rate of up to 50% in infants born with a birth weight of <1 kg. Patent ductus arteriosus is characterized by an abnormal patency of the ductus arteriosus, which diverts blood from the right side of the heart to the systemic circulation during fetal life. Blood typically shunts from the higher-pressured left side (systemic circulation) to the lower-pressured right side (pulmonary circulation). Normally, this ductus closes following spontaneous respiration of the newborn, along with a lowering of endogenous prostaglandin and adenosine levels. The patency is normally sealed with fibrous tissue following a duration of 2-3 weeks. Atrial septal defect is characterized by a shunting of blood from the left atrium to the right atrium, the murmur of which is mild in intensity, located in the pulmonic area, and is associated with a widely-split S2 heart sound. Ventricular septal defect is the most common congenital cardiac malformation. Blood flows from the left ventricle to the right ventricle and presents as a harsh, blowing holosystolic murmur with a thrill localized to the fourth left intercostal space. This murmur may decrease with Valsalva and handgrip. Tetralogy of Fallot is the most common cyanotic congenital heart defect. There are 4 associated defects:

A 33-year-old African American woman with no significant past medical history, who is in her 38th week of a normal pregnancy, presents with a 1-hour history of shortness of breath. She does not recall any precipitating activities or events that may have provoked these symptoms and recalls that she was laying in her bed for several hours since waking when the symptoms developed. She admits to an associated sharp, non-radiating pleuritic chest pain, as well as lower extremity swelling, which she states has been "persistent throughout the course of her pregnancy." She denies palpitations, chest pressure, cough, sputum, fever, chills, changes in weight, rashes, diaphoresis, abdominal pain, nausea, or a history of allergies. Her physical exam is noteworthy for tachypnea and tachycardia, but the rest of the vital signs are normal. Her lungs are clear to auscultation bilaterally, without wheezing, rhonchi, or crackles. Her lower extremities are remarkable for 2+ pitting edema up to the level of her knees; there is no calf tenderness, venous cords, or Homan's sign appreciated. Her skin and mucous membranes were without diaphoresis or cyanosis. A bedside EKG reveals sinus tachycardia at 120 bpm with prominent S waves in lead I and Q waves in lead III. What is the most likely diagnosis? 1 Acute myocardial infarction 2 Bacterial pneumonia 3 Pulmonary edema 4 Pulmonary embolism 5 Bronchial asthma

Correct Answer: Pulmonary embolism Explanation This patient has a presentation most consistent with pulmonary embolism. Nearly all PEs arise from deep venous thrombosis (DVT) in the lower extremity or pelvic veins. Risk factors for DVT and PE are similar in children and adults and include conditions that impair venous return, conditions that cause endothelial injury or dysfunction, and underlying hypercoagulability disorders. Bed rest and confinement without walking, even for a few hours, are common precipitators. Specific risks include an age over 60, atrial fibrillation, cigarette smoking (including passive smoke), estrogen receptor modulators, exogenous estrogens, and progestins (including oral contraceptives and estrogen therapy), extremity or pelvic trauma, heart failure, hypercoagulability disorders, immobilization, indwelling venous catheters, myeloproliferative disorders, nephrotic syndrome, obesity, pregnancy and postpartum states, sickle cell anemia, recent surgeries, and prior venous thromboembolism. Larger emboli cause acute dyspnea, pleuritic chest pain, or both. Dyspnea may be intermittent or occur only with exercise. Less common symptoms include cough and hemoptysis. The most common signs of PE are tachycardia and tachypnea. Less commonly, patients have hypotension, a loud second heart sound (S2) due to a loud pulmonic component (P2), and crackles or wheezing. In the presence of right ventricular failure, distended internal jugular veins and a right ventricular heave may be evident, and right ventricular gallop (third and fourth heart sounds [S3 and S4]), with or without tricuspid regurgitation, may be audible. ECG most often shows tachycardia and various ST-T wave abnormalities, which are not specific for PE. An S1Q3T3 or a new right bundle branch block may indicate the effect of abrupt rise in right ventricular pressure on right ventricular conduction; these findings are moderately specific but insensitive for PE. Pulmonary edema typically causes severe dyspnea, the production of pink and frothy sputum, and diaphoresis and cyanosis. Rales are present in all lung fields, as are generalized wheezing and rhonchi. The chest X-ray usually indicates vascular redistribution, blurriness of vascular outlines, increased interstitial markings, and, characteristically, the butterfly pattern of distribution of alveolar edema. The patient's younger age, quality of chest pain, lack of gastrointestinal and skin findings, and absence of characteristic EKG findings make acute myocardial infarction a less likely diagnosis. The rapid onset of symptoms and denial of fever, cough, and sputum suggest a diagnosis other than bacterial pneumonia, as does the absence of an allergic history and adventitious lung sounds.

A 70-year-old man with type II diabetes mellitus, hyperlipidemia, homocysteinemia, and metabolic syndrome presents with a 5-month history of excessive daytime sleepiness, a lack of refreshing sleep, a depressed mood, and an inability to focus at work and while driving. Additionally, he has been told by his wife that he snores rather loudly while sleeping. He denies fever, chills, headache, cold intolerance, weight loss, hair changes, hoarseness, dysphagia, chest pain, edema, palpitations, or changes in his bowel habits. On physical exam, he is found to be hypertensive. He has elevated BMI with abdominal obesity, and has an enlarged neck circumference; no other abnormalities are noted. Question What pathological mechanism best accounts for this patient's presentation? 1 Bacterial overgrowth of Group A β-hemolytic Streptococcus, causing pharyngeal inflammation 2 Lymphocytic infiltration of the thyroid, causing atrophy and fibrosis of the thyroid follicles 3 Compression of the superior vena cava by an infiltrating mediastinal neoplasm 4 Reduced inspiratory patency of the upper airway due to relaxation of the airway muscles 5 Lower airway inflammation with mucosal thickening and mucus hypersecretion

Correct Answer: Reduced inspiratory patency of the upper airway due to relaxation of the airway muscles Explanation This patient's history and physical exam are remarkable for obstructive sleep apnea/hypopnea syndrome (OSAHS), which is caused by the airway closing on inspiration during sleep. This occurs as the upper-airway dilating muscles relax during sleep. In patients with OSAHS, the dilating muscles fail to oppose negative pressure within the airway during inspiration. Although these patients have narrow upper airways already during wakefulness, increased muscle dilating activity helps to maintain airway patency. However, during sleep, muscle tone falls and the airway further narrows; snoring may commence before the airway occludes, and apnea results. Apneas and hypopneas terminate when the subject arouses (i.e., briefly wakes up). When it occurs in the setting of breast cancer, lung cancer, or lymphoma, the presence of dyspnea; facial, neck, trunk, or extremity swelling; headache; and venous distension, this suggests the possibility of superior cava syndrome. In Hashimoto's thyroiditis, there is a marked lymphocytic infiltration of the thyroid with germinal center formation, atrophy of the thyroid follicles accompanied by oxyphil metaplasia, absence of colloid, and mild to moderate fibrosis. The predominant pathologic event of chronic bronchitis is an inflammatory process in the airways, with mucosal thickening and mucus hypersecretion, resulting in diffuse airflow obstruction.

A 17-year-old boy presents with abdominal pain following a motor vehicle accident. The patient was the passenger in the vehicle and was wearing restraints. He states that he has pain in the left side of his abdomen, which began at the time of the accident. Physical exam reveals a well-developed young man in moderate distress. He has left upper quadrant tenderness, nausea, and positive Kehr's sign. He also has diaphoresis, tachycardia, and hypotension. Question What is the most likely cause of this patient's symptoms? 1 Ruptured abdominal aorta 2 Hepatic injury 3 Splenic injury 4 Ruptured diaphragm 5 Diverticulitis

Correct Answer: Splenic injury The correct response is splenic injury. The spleen is located in the left upper quadrant of the abdomen and is surrounded by the ribs on 3 sides (laterally, anteriorly, and posteriorly). It is in contact with many organs, including the pancreas, diaphragm, stomach, and kidney/adrenal glands. There are several ligaments attached to it, including the splenocolic, splenorenal, gastrosplenic, and splenophrenic. The spleen is often injured through blunt trauma to the abdomen. This can occur from motor vehicle accidents and gunshot, knife, and crush wounds. Injuries on the surface can result in subcapsular hematomas and capsular lacerations. Injury may also reach the internal structures. Injury often results in hemorrhages, which can be small or large. As a result, the patient can have abdominal pain, a positive Kehr's sign (pain in the tip of the shoulder when a person is lying down and the legs are elevated), distension, and may even go into shock due to massive blood loss. Diagnosis of splenic injury is usually by CT scan or ultrasound. Treatment is usually splenectomy. If there is an underlying infection, then splenectomy should be avoided. If the injury is small enough, the patient may be monitored and may not need surgery. A ruptured abdominal aorta would be more likely seen in an older patient with risk factors of smoking history, advancing age, and known vascular disease. Symptoms of presentation would most likely be acute abdominal pain with radiation to back along with a fullness in the abdomen and hypotension. Although hepatic injury is possible with a motor vehicle accident, symptoms would be localized most likely in the right upper quadrant rather than the left abdomen. Patients with a ruptured diaphragm are more likely to present with chest pain rather than abdominal pain. They also will present with respiratory distress which this patient does not have.

A 44-year-old woman G5P5 presents for her annual pelvic examination. Her menses are regular, and she is currently mid-cycle. She notes "leaking urine" when she coughs, sneezes, or strains. There is a bulge into the anterior vaginal wall; it is exacerbated when the patient is asked to "bear down." A urinalysis is unremarkable. There is no cervical motion tenderness or discharge noted on pelvic examination. The patient is afebrile and in no distress. No lesions are noted on the external genitalia and the pelvic examination is unremarkable except for the noted bulge. Vaginal cultures for gonococcus (GC) and chlamydia are pending. Question What is the most likely diagnosis? 1 Bladder infection 2 Menopause 3 Multiple sclerosis 4 Pelvic inflammatory disease 5 Stress urinary incontinence

Correct Answer: Stress urinary incontinence Explanation The patient most likely has stress urinary incontinence as the result of a cystocele. A cystocele is a weakening and stretching of the wall between the bladder and vagina. This condition may cause leakage of urine (especially with coughing, sneezing, or exercising) or incomplete emptying of the bladder. Other names for this condition are "prolapsed bladder" or "fallen bladder." A cystocele is classified as grade 1 when the bladder is prolapsed partway into the vagina. It is grade 2 when the bladder reaches the opening of the vagina. It is grade 3 when the bladder bulges out through the vagina. A cystocele may be caused by childbirth or by excessive straining or lifting. A decreased amount of estrogen occurring during menopause may contribute to a weakening of the muscles that support the bladder and vagina. Treatment options may include avoidance of activities including heavy lifting, Kegel exercises, weight loss, a pessary (a ring-shaped device placed in the vagina that holds the bladder in place), or surgery to create support for the vagina by tightening the muscles and ligaments and repositioning the bladder. A bladder infection could cause symptoms of incontinence. This patient's urinalysis does not suggest infection, and she has not mentioned dysuria.

A 32-year-old man with no significant past medical history presents to his primary care provider with a 2-month history of increased dyspnea upon exertion, which becomes apparent following walking 10 city blocks. He denies any other associated symptoms, such as fever, chills, changes in weight, chest pain, abdominal pain, nausea, or vomiting. He further denies any history of cigarette smoking, occupational risk factors, sick contacts, or recent travel. His physical exam revealed normal vital signs and no distension of his jugular vein. However, there was a prominent right ventricular impulse along the lower-left sternal border associated with a palpable pulmonary artery and a mid-systolic ejection murmur at the upper left sternal border that does not vary in intensity with respiration. There is a fixed split second heart sound. The remainder of his examination is normal. Following diagnostic testing, this patient was referred for surgical repair. Question What is the major long-term complication that requires monitoring following surgical repair? 1 Hypertension 2 Myocardial infarction 3 Mitral valve prolapse 4 Supraventricular arrhythmia 5 Transient ischemic attack

Correct Answer: Supraventricular arrhythmia Explanation This patient's presentation represents an atrial septal defect. The major long-term complication following surgical transcatheter device closure of ASD is the development of supraventricular arrhythmias, although the risk is lowered when the ASD is closed in childhood. The persistence of this risk despite relief of right-sided volume overload is thought to be related to incomplete atrial remodeling or due to the presence of the atriotomy scar. Longer follow-up is required to determine whether device closure alters the risk of atrial dysrhythmias. Generally, short-term complications have included device embolization, aortic root or atrial wall perforation, and cardiac tamponade. Mid- and long-term complications include thrombus formation, device erosion into the aortic root, atrial dysrhythmias, and infective endocarditis. Transient ischemic attacks are associated with untreated atrial septal defects. Patients with ASD who have symptoms related to stroke or transient ischemic attack (especially under 55) or who have hypoxemia should have the ASD closed if no other cause for symptoms is evident.

A 76-year-old man presents with progressive exertional dyspnea associated with substernal chest pain, easy fatigability, and dizziness. Symptoms are exacerbated by walking short distances and relieved with rest. He denies fever, chills, cough, wheezing, pleurisy, calf pain, abdominal complaints, peripheral edema, sick contacts, travel, and cigarette, drug, or alcohol use. His physical exam reveals hypertension and a rough, harsh, low-pitched crescendo-decrescendo systolic murmur beginning after the first heart sound; it is best heard at the second intercostal space in the right upper sternal border. Its intensity is increased toward mid-systole; the murmur radiates to both carotid arteries. It is increased upon squatting and decreased during Valsalva strain. Question Which of the following is correct regarding the management of this patient? 1 β-adrenergic blocker therapy is contraindicated in this patient. 2 An increase in aerobic activity to improve conditioning is recommended. 3 Maximum reduction of preload and afterload with ACE inhibitors is necessary. 4 Surgical intervention provides the only definitive treatment. 5 Bacterial endocarditis prophylaxis is required in patients with this diagnosis.

Correct Answer: Surgical intervention provides the only definitive treatment. Explanation This patient's presentation is significant for aortic stenosis. The only definitive treatment for aortic stenosis is aortic valve replacement. The development of symptoms due to aortic stenosis provides a clear indication for replacement. For patients who are not candidates for aortic replacement, percutaneous aortic balloon valvuloplasty may provide some symptom relief. In cases of moderate aortic stenosis, moderate-to-severe physical exertion and competitive sports should be avoided. Beta-blockers might be used if the predominant symptom is angina. The medical treatment options are limited in symptomatic patients with aortic stenosis who are not candidates for surgery. Those with pulmonary congestion may benefit from cautious use of digitalis, diuretics, and angiotensin-converting enzyme (ACE) inhibitors. Although vasodilators may be used for heart failure and for hypertension associated with aortic stenosis, they should be used with extreme caution in order to prevent excessive decreases in preload or systemic arterial blood pressure. Reducing the blood pressure to normal levels in patients with hypertension and aortic stenosis is advisable, but hypotension must be avoided. Antibiotic prophylaxis for the prevention of bacterial endocarditis is no longer recommended in patients with valvular aortic stenosis.

A 68-year-old man with a past medical history of hypertension, hyperlipidemia, cluster headaches, polymyalgia rheumatica, and type 2 diabetes mellitus presents due to a 2-day history of constant left-sided throbbing headache of moderate severity. He admits to associated symptoms, such as pain across his mandible when he eats, fever, fatigue, and muscle aches. Most alarming to the patient was a single episode of complete left eye blindness that lasted for 30 minutes but has since resolved. He denies extremity numbness, tingling, muscle weakness, incontinence, and changes in mental status. He further denies rhinorrhea, ocular discharge, nausea, and vomiting. The physical exam was remarkable only for a tender left scalp with a noticeable pulsation underlying the tender area. Question What is the most likely diagnosis? 1 Bell's palsy 2 Herpes zoster ophthalmicus 3 Cerebrovascular accident 4 Cluster headache 5 Temporal arteritis

Correct Answer: Temporal arteritis Explanation This patient most likely has temporal arteritis. This patient has a past medical history of polymyalgia rheumatica, which portends a risk of clinical temporal arteritis of 10-15%. Symptoms include headache localized to the arteries of the scalp; polymyalgic symptoms (constitutional) may occur in over 50% of patients. Cranial artery tenderness, masseter claudication, and amaurosis fugax are all common manifestations. The presence of moderately severe persistent headache and absence of facial drooping, muscle paresis, drooling, and eyelid weakness reliably exclude the diagnosis of Bell's palsy. The lack of dermatomal findings on physical exam consistent with Herpes zoster (grouped vesicles on an erythematous base in a dermatomal distribution) and prodromal pruritis, numbness, burning, and tingling make herpes zoster unlikely. Although this patient has strong risk factors that may contribute to ischemic stroke development, additional stroke manifestations beyond hemianopia, such as hemisensory loss, hemiparesis or paralysis, dysarthria, aphasia, ataxia, vertigo, or changes in consciousness, are not experienced here. The typical descriptive features of cluster headaches are an intense non-throbbing unilateral retro-ocular headache that is stabbing or burning and accompanied by ipsilateral lacrimation, nasal stuffiness, and facial flushing. Ipsilateral ptosis and miosis may also be present.

A 29-year-old Caucasian man with a prior history significant for right cryptorchidism presents for a routine physical examination. His prior cryptorchidism was corrected by orchiopexy at the age of 6 months. He has no complaints at present. His physical exam reveals no abnormalities other than bilateral gynecomastia and a right testicular mass that is painless and firm; it measures approximately 1.5 cm in diameter. The right testicular mass does not transilluminate, nor does it disappear when the patient lies supine. There is no femoral or inguinal lymphadenopathy, nor are there palpable hernias. Question What is the most likely diagnosis? 1 Testicular malignancy 2 Scrotal hydrocele 3 Epididymitis 4 Orchitis 5 Varicocele

Correct Answer: Testicular malignancy Explanation This patient's physical exam findings suggest testicular malignancy. In the United States, the incidence of testicular cancer in African Americans is approximately 1/4 of that in Caucasians. Within a given race, individuals in the higher socioeconomic classes have approximately twice the incidence of those in the lower classes. Testicular cancer is slightly more common on the right side than on the left, coinciding with higher rates of right cryptorchidism. Of all acquired and congenital risk factors, cryptorchidism is the strongest associated risk factor associated with testicular cancer. Placement of the cryptorchid testis into the scrotum (orchiopexy) does not alter the malignant potential of the cryptorchid testis, but it does facilitate examination and tumor detection. The most common symptom of testicular cancer is a painless enlargement of the testis. Enlargement is usually gradual, and a sensation of testicular heaviness is not unusual. Acute testicular pain is seen in approximately 10% of cases and may be the result of intratesticular hemorrhage or infarction. Approximately 10% of patients are asymptomatic at presentation, and the tumor may be detected incidentally following trauma or by the patient's sexual partner. On physical exam, a testicular mass or diffuse enlargement is found in most cases. The mass is typically firm and nontender, and the epididymis should be easily separable from it. A hydrocele may accompany the testicular tumor and help to camouflage it. Transillumination of the scrotum can help to distinguish between these entities.

A 10-year-old boy presents with a 1-week history of a rash on his scalp. His mother states that the boy has been scratching his head often, and she notes that there are areas where his hair appears to have fallen out. She attempted to treat it with over-the-counter preparations, but his condition has not improved. The boy is active and otherwise healthy. The rash appears as erythematous, circular, scaly patches. There are areas where the hairs have become brittle and broken off. A scraping of one of the patches is placed in potassium hydroxide solution and shows hyphae. Question What is the most likely diagnosis? 1 Vitiligo 2 Tinea versicolor 3 Psoriasis 4 Tinea capitis 5 Seborrheic dermatitis

Correct Answer: Tinea capitis Explanation Tinea capitis, also known as ringworm, is a fungal skin infection that is more common in children. It is contagious and can be difficult to rid the patient of the infection. It is called ringworm due to the appearance of the infection on the skin; the lesions produced are ring-shaped and were once thought to be caused by a worm. It is caused by a fungus from the species Trichophyton and Microsporum. The lesions usually begin as round, often reddened, papules on the scalp. Over the course of a few days, the papules become scaly and coalesce to form a ring-like shape. The hair often breaks off in the affected area, making the infection more noticeable. There can be itching, swelling, and occasionally a purulent discharge. Diagnosis can be confirmed by microscopic analysis. A scraping of the area placed in potassium hydroxide solution will show hyphae if a fungal infection is present. Once this has been established, treatment can begin. Tinea capitis is treated with oral antifungal medications; topical preparations are usually not very effective. Griseofulvin, itraconazole, and terbinafine are most commonly used. Vitiligo causes a loss of pigmentation. Tinea versicolor is common in adults and adolescents. The most commonly affected areas include the chest, back, and shoulders. Psoriasis causes raised, scaly patches that do not fluoresce under UV light. Seborrheic dermatitis causes patches of thick, scaly, crusty skin; it is found mainly on the scalp. It can also appear in areas with oil glands (e.g., folds of skin, like those of the face, groin, and armpits).

A 38-year-old woman with a past medical history of rheumatic fever and endocarditis presents with progressive dyspnea on exertion associated with palpitations and intermittent episodes of left-sided chest pain. Both symptoms resolve at rest. Her physical exam reveals resting tachycardia and a widened pulse pressure. The cardiac exam is notable for a decrescendo, diastolic, high-pitched murmur, which is loudest at the left sternal border and accentuated with the patient leaning forward in full expiration. Abrupt distention and quick collapse are observed upon palpation of the peripheral arterial pulses, and booming systolic and diastolic sounds are auscultated over the femoral arteries. Question What is the diagnostic test of choice to confirm this patient's most likely diagnosis? 1 Transthoracic echocardiography 2 Chest radiography 3 Cardiac computed tomography 4 Electrocardiography 5 Cardiac troponins

Correct Answer: Transthoracic echocardiography Explanation This patient's most likely diagnosis is aortic regurgitation. Patients exhibit tachycardia and widened pulse pressures. The classic auscultatory finding is a decrescendo, diastolic, high-pitched murmur loudest at the left sternal border that is accentuated with the patient leaning forward in full expiration. An Austin-Flint murmur may be evident, characterized by a low-pitched, mid-diastolic rumbling murmur due to blood jets from the murmur striking the anterior leaflet of the mitral valve, which results in premature closure of the mitral leaflets. Corrigan pulse ("water-hammer" pulse) is described as an abrupt distention and quick collapse on palpation of the peripheral arterial pulse; booming systolic and diastolic sounds auscultated over the femoral arteries represent Traube sign ("pistol-shot" pulse). Transthoracic echocardiography (TTE) should be performed in all patients with suspected aortic regurgitation and should be performed periodically in patients with confirmed disease of significant severity. TTE is a highly accurate test in aortic regurgitation with sensitivity and specificity well in excess of 90%. Its parameters are used to determine the optimal timing of surgery in many cases. Chest radiography may show evidence of structural abnormalities, such as aortic dilation, prosthetic valve dislodgement, aortic valvular calcification, or functional compromise (pulmonary edema, cardiomegaly). Cardiac computed tomography (CT) scanning (along with magnetic resonance imaging) have not yet achieved widespread adoption in the management of aortic regurgitation. Electrocardiographic findings in aortic regurgitation are nonspecific but may reveal left ventricular hypertrophy, left axis deviation, left atrial enlargement, left ventricle volume overload pattern, and conduction defects. Cardiac troponins are not expected to be of value since this patient is not undergoing cardiac necrosis and is presently asymptomatic.

A 50-year-old man with a past medical history of hyperlipidemia presents with a recurrent "bitterness and sour sensation" in his mouth and a nonproductive cough; both have been occurring over the last year. There is associated progressive dysphagia and odynophagia to solid foods, but not to liquids; as a result, he has lost 15 lb over the past 5 months. He admits to drinking 1-2 beers per week, but he denies additional alcohol use. He also denies cigarette smoking and illicit drug use. He denies fever, chills, sputum production, chest pain, palpitations, dyspnea, shortness of breath, wheezing, peripheral edema, abdominal pain, nausea, vomiting, diarrhea, melena, hematochezia, or changes in bowel habits. The physical exam reveals normal vital signs. Oropharyngeal, neck, cardiopulmonary, and abdominal exams are normal. Question What is the most appropriate initial intervention for this patient at this time? 1 Chest X-ray 2 Barium esophagography 3 Trial prescription of esomeprazole 4 Upper endoscopic evaluation 5 Ambulatory esophageal pH monitoring

Correct Answer: Upper endoscopic evaluation Explanation An important part of the medical history of dysphagia is to determine the type of food that produces the symptoms. (solids, liquids, or both). If the patient is without a history of radiation, caustic injury, esophageal cancer, or complex stricture, an upper endoscopy with or without esophageal biopsies would be the initial test to determine the cause of progressive esophageal dysphagia to solids. Mucosal inspection and the ability to biopsy is more precise via this modality. Dilation can be performed during the study if needed, so endoscopy may also potentially serve as a therapeutic tool. Alternately, a barium esophagography provides more information regarding motility disorders. A video-modified barium swallow is the initial diagnostic test of choice for oropharyngeal dysphagia, a disorder of motility. Although a trial prescription of esomeprazole may be suitable for a more straight forward case of simple GERD, the patient has multiple risk factors that puts him at a higher risk of an incidence of malignancy, warranting a more confirmatory diagnostic scenario at this time. A chest X-ray may reveal adenopathy, a widened mediastinum, metastatic lesions to the lungs or bone, or signs of tracheoesophageal fistula, such as pneumonia. It will not, however, reliably identify esophageal lesions. Ambulatory esophageal pH monitoring is the preferred study for documenting acid reflux. It is indicated in the following clinical scenarios: for patients being considered for anti-reflux surgery with a normal endoscopy in patients with a normal endoscopy but with continued reflux symptoms following therapy with a proton pump inhibitor in the evaluation of atypical reflux symptoms that continues despite anti-reflux therapy Patients with alarm symptoms, such as unexplained weight loss, odynophagia, jaundice, recurrent vomiting, blood loss, a palpable mass or lymph node, or a family history of GI malignancy, should have diagnostic testing performed promptly.

A 28-year-old woman with a past medical history of well-controlled asthma presents with recurrent sneezing episodes, nasal itching, congestion, and headache. Her physical exam reveals post-nasal drip, a transverse nasal crease, and bilateral infraorbital cyanosis. Question What additional finding supports an allergic etiology of this patient's presentation? 1 Associated manifestations, including malaise, body aches, and cough 2 Pale bluish nasal mucosa upon speculum examination 3 Provocation by changes in temperature or exposure to odors and chemicals 4 An erythematous and edematous nasal septum and turbinates 5 The presence of fever and copious purulent green nasal discharge

Pale bluish nasal mucosa upon speculum examination Explanation This patient's past medical history and current presentation are remarkable for allergic rhinitis. Signs and symptoms include sneezing paroxysms, clear rhinorrhea, nasal congestion, pale bluish nasal mucosa, transverse nasal crease, infraorbital cyanosis (allergic shiners), serous otitis media, along with nasal, ocular, or palatal itching. Viral rhinitis is often associated with other manifestations of viral illness, which can include headache, malaise, body aches, and cough. Nasal drainage in viral rhinitis is most often clear or white and can be accompanied by nasal congestion and sneezing. The nasal septum and turbinates are typically erythematous and edematous. Patients with vasomotor rhinitis present with symptoms of nasal obstruction and clear nasal drainage. The symptoms are often associated with changes in temperature, eating, exposure to odors and chemicals, or alcohol use. Some clinicians suggest that abnormal autonomic regulation of nasal function leads to vasomotor rhinitis. The presence of fever in conjunction with copious, purulent green nasal discharge should raise suspicion for underlying infection, especially bacterial rhinosinusitis.

A 45-year-old woman with no significant past medical history presents with a 4-month history of a dull, aching heaviness sensation in her proximal right leg. She notes that this sensation is provoked by extended periods of standing and walking and is relieved when she lies in a recumbent position. Her past medical history is remarkable for pregnancy 4 times, the most recent being approximately 2 years ago. She denies a history of smoking, trauma, injuries, fever, chills, chest pain, shortness of breath, hemoptysis, cough, skin changes and coolness, and peripheral edema. Her physical exam reveals several dilated, tortuous, elongated veins along the medial right thigh, which are especially pronounced upon standing. The remainder of the physical exam is normal. What is the most appropriate therapeutic approach for this patient at this time? 1 Warfarin (Coumadin) 2 Clopidogrel (Plavix) 3 Sclerotherapy 4 Furosemide (Lasix) 5 Cilostazol (Pletal)

Sclerotherapy Explanation This patient's diagnosis is varicose veins. Sclerotherapy can be used to treat varicose veins. It involves the injection of an irritating solution into the varicose vein to promote an inflammatory response, scarring, and obliteration of the lumen. Warfarin is indicated for myocardial infarction or cerebrovascular accident prevention and in patients with atrial fibrillation, mechanical heart valves, or deep venous thrombosis. Diuretics are ineffective for varicose veins. Cilostazol and clopidogrel both inhibit platelets. Clopidogrel is used in CAD and CVA prevention, as well as post-ST-segment MI. Cilostazol inhibits phosphodiesterase III which inhibits platelet aggregation and has effects on vasculature. It is indicated in treatment of intermittent claudication.

A 67-year-old man with an 80-pack/year smoking history and chronic bronchitis presents 30 minutes after suffering what was described as a "seizure" by his wife. According to his wife, this was his first seizure. During the seizure episode, the patient did not lose consciousness, but was observed to have developed an "involuntary jerking in his right arm and leg" that lasted about 5 minutes, with right arm weakness immediately following this involuntary activity. On physical exam, the patient appears drowsy, confused, and answers questions slowly. His right arm has objective weakness rated as 4/5; the remainder of the neurological exam is normal. Question What aspect of this patient's presentation most closely identifies this event as a simple seizure? 1 Involuntary extremity jerking 2 Right arm weakness is noted 3 There was no impairment of consciousness 4 There is postictal confusion 5 First seizure as an adult

There was no impairment of consciousness Explanation Focal seizures without dyscognitive features (formerly termed simple partial seizures) can cause motor, sensory, autonomic, or psychic symptoms without impairment of cognition. Involuntary movements are typically clonic (repetitive, flexion/extension movements). Other possible manifestations include somatosensory symptoms (e.g., paresthesias or tingling) that spread (or "march") to different parts of the limb or body depending on their cortical representation. Generalized tonic-clonic seizures are attacks in which consciousness is lost. A focal seizure may progress to a generalized seizure, in which loss of consciousness occurs secondarily. The initial phase of generalized tonic-clonic seizures is usually tonic contraction of muscles throughout the body. Contraction of the jaw muscles may cause biting of the tongue. A marked enhancement of sympathetic tone leads to increases in heart rate, blood pressure, and pupillary size. After 10-20 seconds, the tonic phase of the seizure typically evolves into the clonic phase, produced by the superimposition of periods of muscle relaxation on the tonic muscle contraction. The postictal phase is characterized by unresponsiveness, muscular flaccidity, and excessive salivation that can cause stridorous breathing and partial airway obstruction. Bladder or bowel incontinence may occur at this point.


Conjuntos de estudio relacionados

QUIZ 9b. The Water-Soluble Vitamins: B Vitamins and Vitamin C

View Set

Intro to Business Chapters 5, 6, and 8

View Set

Texas Promulgated Contract Forms: Ch. 6 Quiz

View Set

Week 2 APM Study Guide - De La Fuente

View Set

Chapter 13 - Spinal Control of Movement

View Set

Chapter 16: Care of Postoperative Patients

View Set

Promotion Board Study Guide Customs and Courtesies

View Set